Neuro

Réussis tes devoirs et examens dès maintenant avec Quizwiz!

MOA of Digoxin?

Digoxin inhibits Na-K+ ATPase pump in myocardial cells. Results in high intracellular Na+ levels than normal. This in turn will indirectly inhibit the Na+-Ca2+ exchanger (which would inhibit Na+ from coming in and Ca2+ from going out of cell) Increased intracell. Ca2+ now allows for stronger heart contractility. Helps pts with CHF.

Drugs that actually cause drug-mediated toxic effects directly on nerves causing neuropathy?

Direct toxic effects to nerves occur with Chemo drugs like Microtubule inhibitors (Vincristine, Paclitaxel)

What med is an AchE inhibitor used to treat Alzheimer's Disease?

Donepezil is an AchE inhibitor used to treat Alzheimer's Disease.

man comes in to office with "restless sleep" and has strange sensation in legs when trying to sleep. Feeling goes away when he stretches legs or gets out of bed. Best tx?

Dopamine agonist (Ropinirole, Pramipexole ) Key: pt has classic Restless leg syndrome, worse at rest/sleep. Gets better with Dopamine agonists

Why do kids with Down Syndrome have high risk of later getting Alz Dx?

Down's Syndrome has 3 copies of Chromosome 21 (contains APP) which increases Beta-Amyloid plaques and Neurofibrillary tangles in brain.

4 month old boy with hereditary metabolic disorder, presents with FTT. Born outside U.S., and has had severe seizures. Abnormal shaped head, with large anterior Fontanelle. Profoundly hypotonic, with hepatomegaly. Labs: Elevated VLCFAs, with impaired ability to oxidize these VLCFAs. Dysfunction of what cell structure?

Dysfunction of Peroxisomes Key: This baby boy has a buildup of VLCFAs and branched chain fatty acids due to inability of his dysfunctional Peroxisomes to allow for oxidation of the VLCFAs and branched-chain fatty acids. Key: This is called: X-Linked Adrenoleukodystrophy Accumulation of these fatty acids in brain causes permanent neurological dysfunction and death.

When would Protoporphyrin be high?

Elevated Protoporphyrin occurs in: 1. Iron deficiency anemia 2. Lead (Pb2+) poisoning

Risk of s/p Embolic stroke?

Embolic ischemic strokes have very high risk of undergoing an acute hemorrhage within 7 days following ischemic event!

What is "endomysial" inflammatory infiltrate characteristic of?

Endomysial inflammation occurs in classic: Poliomyositis (inflammation of skeletal muscles due to unknown cause) S+S of Poliomyositis: Bilateral, symmetric muscle weakness (in Proximal muscles) Note: Reflexes (DTR) are normal, bc Poliomyositis only affects muscles, not nerves.

32 y/o man has tinging in feet, now in knees. Weakness started 1 week ago, now getting worse. Had a mild respiratory tract infection 2 weeks ago. What is his condition associated with?

Endoneural inflammatory infiltration Key: pt has classic GBS (C. jejuni). Causes acute demyelinating peripheral neuropathy, via demyelination of peripheral nerves. This causes "endoneural inflammatory infiltrate," leading to ascending paralysis.

5 y/o boy with Generalized tonic-clonic seizures for past 24 hours. In office, he is having sustained seizure. MOA of most appropriate drug?

Enhanced postsynaptic Chloride influx Key: Pt needs an IV of Benzos (Lorazepam) as #1DOC for status epilepticus. Benzos will increase GABA effect and also frequency of Cl- influx, which would hyperpolarize and inhibit action potential.

Communicating Hydroceph?

Enlargement of ventricles due to excess CSF due to lack of Absorption Key: CSF flows freely between ventricles, but lack of absorption via Arachnoid granulations. Types of Communic. Hydroceph 1. Nomral pressure hydroceph: occurs in old people. Ventricles slowly enlarge due to lack of absorption. Wet, wobbly, wacky. 2. Thrombosis, meningitis, SAH, can all impair CSF absorption, causing Communic Hydroceph. S+S: change in mental status, seizure, focal neuro deficits.

25 y/o woman with new-onset seizures comes to ER with fever, skin rash. Had her first seizure 6 weeks ago, and MRI of brain shows lesion. She was on Pheytoin 4 weeks ago. Temp is 102. Erythema over 60% of body, lymphedema, symmetrical facial swelling. What lab finding?

Eosinophilia Key: Drug-induced Eosinophilia occurs after exposure to "high risk" drugs like anticonvulsants (Phenytoin, Carbamazepine) or Allopurinol or Sulfonamides or Vancomycin, etc. Hallmark: fever, lymphedema, facial edema, skin rash, Eosinophilia, organ failure!!

Where do epyndomas arise from?

Epyndomas are paraventricular tumors arising from floor of 4th ventricle. These are the ones that have "perivascular rosettes"

MOA of #1DOC for absence seizures?

Ethosuximide is the #1DOC for Absence seizures. MOA: Ethosuximide blocks T-type Ca2+ channels and decreases Ca2+ current in Thalamic neurons. #1DOC for absence seizures.

Common bone tumor in older kids/teens, especially in Long Bones?

Ewing Sarcoma is a malignant Long bone tumor in kids/teens (can present like acute osteomyelitis)

MOA of Prion Dx?

Ex of Prion Dx (CJD, Kuru (Cannibals eat human brain), and fatal familial insomnia) MOA: Abnormal prion protein (PrP) in host neurons, resistant to proteases/ubiquination Causes Spongiform encephalopathy with cysts in brain

What is a CytP450 inducer?

Ex: Phenytoin is a CytP450 inducer. This means it increases the metabolism of other meds that are metabolized by the liver, like OCPs. Key: When CytP450 is induced, serum conc of other meds goes down, bc of increased metabolism. reducing "efficacy."

The brachial plexus controls all motor activity of the UEX, except for what muscles?

Except for The Trapz and SCM (innervated by CN 11 Accessory) SCM: flexes and rotates head/neck

Exons vs Introns?

Exons are proteins that code for DNA. Introns are removed during intranuclear mRNA processing, and not present in final mRNA sequence.

Examples of Exotoxins?

Exotoxins include: 1. S. aureus TSS-1 exotoxin 2. S. aureus enterotoxin (food poisoning exotoxins) 3. Botulism exotoxins (C. botulism) 4. C. diphtheria exotoxin 5. C. Tetani exotoxin 6. Bordatella pertussis exotoxin (Whooping cough caused by the exotoxin)

Pick's Disease

FTLD is Pick's Disease S+S: personality and behavior changes, inappropriate disinhibition, abnormal speech (aphasia)

Which sphingolipid dx causes accumulation of Globotriaosylceramide?

Fabry's Dx S+S: Globotriaosylceramide can't be converted to Glucocerebroside, due to defect in Alpha-galactosidase A Causes Angiokeratomas, R.F., and peripheral neuropathy. Note: S+S occur in adulthood

3 wk history of difficulty hearing. Can't tolerate everyday sounds, complains of ear pain, avoids public places. What CN?

Facial CN7 Key: Pt has Stapedius muscle paralysis. This muscle is innervated by Stapedius nerve (branch of CN7). Paralysis leads to hyperacusis (increased sound sensitivity)

36 y/o woman with swelling of right cheek, fullness of preauricular space on Right side. MRI shows 2.2 cm mass in Right parotid gland, which biopsy shows is neoplastic. If left untx, pt will develop what?

Facial droop Key: CN7 courses through Parotid gland, where it divides into 5 terminal branches to innervate muscles of facial expression. Malignant Parotid gland tumors compress and disrupt facial nerve, causes Ipsilateral Facial droopie droop. CN7 Palsy: 1. loss of forehead/brow movements 2. can't close eye, droopy eyelid 3. loss of nasolabial folds and drooping of lower lip.

When does Fat Necrosis occur?

Fat necrosis occurs for example in Acute pancreatitis. Release of premature pancreatic enzymes (lipases) causes autodigestion of adipose cells, and release of fatty acids. Key: Saponification (chalky-white deposits), form when fatty acids saponify/combine with Calcium

Describe the femoral ring?

Femoral ring is the upper opening of the femoral canal. Femoral canal: contains lymphatic vessels/lymphatic nodes Key: Femoral nerve lies laterally to femoral artery and femoral canal. Femoral nerve IS NOT in femoral sheath N(AVEL)

What medication is used to treat a Benzodiazepine overdose?

Flumazenil (a GABA receptor antagonist)

Phalen's Sign

For Carpel Tunnel Syndrome (flexion of wrists) reproduces S+S

What happens with injury to CN V3? Mandibular branch of Trigeminal CN 3 injury on one side: causes unopposed action of Opposite jaw's Pterygoids. Leads to deviation of jaw Towards side of injury.

Foramen Ovale (CN V3)

Fracture of Surgical neck of Humerus would damage what nerve?

Fracture to Surgical neck of Humerus/Anterior dislocation of GH joint causes: Axillary nerve damage. S+S: paralysis of Deltoid and teres minor (weak arm ABduction)

35 y/o pregnant woman has amniocentesis, reveals increased levels of AchE. Amnio suggests failure of what process?

Fusion of the edges of the neural plate Neural tube defect due to failure of fusion of neural plate during 4th week gestation. NTD cause: leakage of AFP (alpha-fetoprotein) and AchE into amniotic fluid. Key: Detection of AFP and AChE is used for prenatal screening of NTDs (Spina bifida cystica, anencephaly)

What is GABA formed from?

GABA (Gamma-aminobutyrate) is formed from Glutamate decarboxylation

8 month old girl has irritability, regression of motor skills. Can no longer sit, or roll over, roll over. Startles easily with koud noises. Macrocephaly, bright red fovea centralis with contrasted white macula. What metabolite is accumulated in excess in this baby girl?

GM2 ganglioside Key: Pt has classic Tay Sachs disease (A.R.) caused by Beta-hexoaminidase A deficiency. Results in excess GM2 ganglioside accumulation. S+S of Tay Sachs: Progressive neurodegeneration, regression, cherry-red macula spots (can cause blindness), and macrocephaly. Tay Sachs is most common in Ashkenazi Jews, and pts die by 3-5 y/o

Anti-epileptic medication that blocks voltage-gated Calcium channels?

Gabapentin is an anticonvulsant that blocks Voltage gated Calcium channels, which inhibits fusion and release of neurotransmitter vesicles into synaptic cleft. Seizures are abnormal, synchronized firing of hyperexcitable neurons in brain. Anti-seizure drugs work to modify/inhibit electrochemical transmission between neurons.

3 y/o boy has M.R. and speech delay. No social smile, no interest like other kiddies. Only has 8-10 word vocab, and doesn't combine 2 word phrases. Poor verbal comprehension too. Labs show 226 CGG trinucleotide repeat on X chromosome. Cause of condition?

Gene methylation Key: pt has Fragile X Syndrome due to CGG trinucleotide repeat on FMR1 gene (Fragile X mental retardation gene) on long arm of X chromosomes. This process causes classic hypermethylation and INACTIVATION of FMR1 gene.

What is Phenobarbital used for?

Generalized tonic clonic seizures

Most common location for Germ cell tumor?

Germ cell tumors arise from Pineal gland Can cause Obstructive hydrocephalus, Increased ICP (headache, N/V, altered mental status)

4 day old premie in NICU with hypotonia. Lethargic with weak, high-pitched cry. Blood in lateral ventricles. What structure is source of blood?

Germinal matrix Key: preemie babies commonly hemorrhage out bc of a fragile germinal matrix. (neonatal intraventricular hemorrhage)

60 y/o farmer brought to ER due to confusion, muscle cramps, and difficulty breathing. P.E. shows excessive sweating, wheezing, and bradycardia. Pupils are constricted, symmetric, and reactive to light. What med should be given, and also what is the pt still at risk for?

Give the pt stat IV Atropine This farmer has classic organophosphate poisoning, which causes way too much excess stimulation of Muscarinic and Nicotininc Cholinergic receptors. Atropin will reverse these excess PNS effects, by being an M3-Muscarininc receptor antagonist. Key Key: However, Atropine will NOT help reverse the muscle cramps/muscle paralysis (Nicotinic)

62 y/o woman with ST-segment elevation on ECG is given a medicine which helps treat the bradycardia. However, she now complains of severe right eye pain. Why?

Glaucoma Key: ST-segment elevation in inferior leads is hallmark of: Inferior MI (due to blockage of Right coronary artery which supplies SA and AV node). Inferior MIs cause bradycardia. Pt was given Atropine for treatment of Bradycardia, since it decreases vagal influence on SA and AV nodes. Common side effect is increased IOP (causing increased Glaucoma)

Greater than 2 weeks s/p I.S.?

Glial scar forms Can form a cystic space in about a >1 month

Which brain tumor is highly aggressive, deadly, and has bizarre pseudopalisading rosettes.

Glioblastoma

Most common primary malignant tumor?

Glioblastomas are the most common primary malignant brain tumor in adults. Located in cerebral hemispheres, and can cross corpus callosum. Glioblastomas are "butterfly gliomas" with the "pseudopalisading rossettes"

64 y/o alcoholic homeless man has abdominal pain in hospital with elevated Amylase and Lipase. In hospital, develops acute confusion, horizontal Nystagmus and lesions in Mamillary bodies. Why did he develop abdominal pain in hx?

Glucose infusion Infusion of Glucose without the Thiamine (Vit B1) in pts with Thiamine deficiency causes encephalopathy, and worsening of the CAN. Key: Thiamine is a cofactor needed for glucose metabolism. Infusion of glucose without Thiamine will exacerbate the pre-existing Thiamine deficiency.

3 wk old baby with fever, chills, high Neutrophil count. Blood culture shows Gram negative rods that form Pink colonies on MacConkey agar. Most important virulence factor?

Gram negative, rod, Lactose Fermenter on MacKonkey Agar = either E. coli or Klebsiella, Serratia, Enterobacter. It' s E.coli causing Gram - meningitis in newborns (with capsule with K antigen) Virulence factor: Capsule

How does LPS/LOS cause septic shock?

Gram- bacteria LPS/LOS causes sepsis by triggering a ton of systemic inflammatory cytokines. Inflammatory cytokines: TNF-alpha, IL-6, IL-8 via interaction of LPS with the Toll-like receptors (TLRs) Sepsis causes the hypotensive, tachycardia--->shock--->death

Segmental demyelination of peripheral nerves, due to a T-cell response. Dx?

Gullain-Barre Syndrome Key: GBS is an immune-mediated acute demyelination of peripheral nerves. C. jejuni. Causes ascending paralysis. Key: death usually occurs due to paralysis of respiratory muscles

Labs of HHC?

HHC is an A.R. disease Due to lack of enough Transferrin (to transport the Iron), labs show Labs: Elevated serum Ferritin, increased Saturation of what little Transferrin there is. Causes increased intestinal Iron absorption, liver failure, and "Bronze Diabetes," as well as Dilated Cardiomyopathy

What is Haloperidol good for?

Haloperidol helps treat the +Positive S+S of Psychosis (Delusions, Hallucination)

subependymal hamartomas are seen in?

Hamartomas are seen in TS (Tuberous Sclerosis) TS causes cysts in liver, kidney, and hamartomas in CNS. Pimple looking things (benign hamartomas) in T-zone of face. Renal angiomyolipomas Worst S.E.: seizures

Carotid artery artherosclerosis and Cardiac emboli can cause?

Heart attack (M.I.) and Acute Ischemic Stroke Causes sudden-onset neuro deficits that are maximal at symptom onset.

22 y/o male brought to ER with knife wound in neck after bar fight. Right-sided hemiparesis, with loss of proprioception and loss of vibration sense BELOW C8 dermatome on RIGHT. On LEFT side, has loss of pain and temp BELOW T2 dermatome. MRI of spine reveals what?

Hemisection of RIGHT side of the spinal cord, at C8 nerve root (C7) Key: This is classic Brown-Sequard Syndrome, which results in Hemisection of Spinal Cord. S+S: 1. IPSILATERAL Spastic paralysis (due to damage of CST) 2. IPSILATERAL loss of tactile, vibratory, and position sense (DCT) 3. CONTRALATERAL loss of pain+temp (STT) all occurs BELOW level of injury.

Difference between Heterchromatin and Euchromatin?

Heterchromatin is highly condensed chromatin that's methylated DNA (inactivated) and has a low level of transcriptional activity. Euchromatin has very high levels of transcription. Note: Histone acetylation promotes formation of Euchromatin.

12 y/o with seizure disorder experiences several strokes, causing neurological deficit, muscle weakness. Blood tests show increased serum Lactate both at rest and post-exercise. Maternally inherited condition. Patient's sister is also affected with SAME disorder, but displays vey few S+S. Why?

Heteroplasmy This kid has MELAS Key: Mitochondrial diseases have exclusively-maternal inheritance. Variable severity of these diseases is explained by random distribution of normal and mutated mitochondria between daughter cells during mitosis. So; some cells may have completely healthy mitochondria, while other cells have mitochondria affected by genetic mutation, so S+S vary from pt to pt. (Aka Heteroplasmy)

In clinical experiment, a new inhaled anesthetic (Drug A) is tested. Partial pressure in arterial blood is shown as a function of time after being inhaled. Compared to similar curve with NO. Both drugs admin at same Partial Pressure. Drug A's Partial Pressure in blood goes down much faster than NO. Why?

High blood/gas partitioning coefficient Key: Onset of action of an inhaled gas anesthetic depends on its: Solubility in blood (aka Blood/gas partition coefficient) Key: So, drugs with very high Blood/gas partition coefficient are highly soluble in blood. High blood solubility, however, means slower equilibration with brain, so its takes a longer time for it to reach brain and start working.

65 y/o man has sudden onset weakness and difficulty speaking. Takes Warfarin for his PMH with A-Fib. His wife states that he's now taking 2 new drugs as well. P.E. shows Right Hemiplegia, hemisensory loss, Expressive aphasia, and Right Homonymous Hemianopia. Cause?

His "new drug" he's taking is St. Johns Wart Key: St. Johns Wart is a CytP450 inducer. This means it speeds up the metabolism of drugs that are normally metabolized by CytP450 in Liver. Net Effect of a CytP450 Inducer: Drugs like Warfarin (that get metab by CytP450) will have a lower plasma concentration and decreased efficacy. Since Warfarin isn't able to do it's job, pt suffered from a Thromboembolic stroke

25 y/o male with suspected tetanus. Suffered a minor LEX wound a week ago. Most important in making the diagnosis?

History and physical exam Key: Diagnosis of Tetanus is Clinical. You know he suffered a wound but you have to ask from what? Metal? Also have to find out if he's vaccinated. S+S: Trismus, Rhesus Sardonicus smile, arched back, muscle spasms, trismus ("lockjaw") Note: Serum toxin assay or Blood cultures would be useless bc Tetanus is due to a toxin released from a local wound infection.

Causes of Horner's Syndrome?

Horner's (ptsosis, miosis, anhydrosis) can occur due to interruption of SNS innervation to head. 1. Lesion to Lateral Hypothalamus 2. Lesion to Paravertebral SNS chain/Stellate Ganglion (Pancoast Tumor)

When does Horner's Syndrome occur?

Horner's occurs (ptosis, miosis, anhydrosis) due to lesion of: 1. Ipsilateral Lateral Hypothalamus 2. or Lesion of SNS fibers in Brainstem (Medulla) on Ipsilateral side 3. or From Pancoast tumor compressing the Cervical SNS ganglion

Where do you see hyalinization of nerve arterioles?

Hyalinization of the nerve arterioles occurs with D.M. (Diabetic neuropathy) Leads to narrowing and occlusion of arterial lumen, and ISCHEMIC nerve damage (lack of enough blood flow) Also, Diabetic neuropathy can occur due to accumulation of Sorbitol and Fructose in nerve axons.

Risk of Global ischemic hypoperfusion?

Hypoxic ischemic encephalopathy occurs due to Global interruption of Cerebral Blood Flow Occurs in Cardiac Arrest Net effect: coma, vegetative state

45 y/o smoker with HTN and DM. Severe substernal chest pain. ECG shows ST-elevation (M.I.) Pt dies from A-Fib. Autopsy shows Wedge-shaped strips of necrosis over the cerebral convexity, parallel to longitudinal cerebral fissures. Cause?

Hypoxic-ischemic encephalopathy Key: Profound cerebral hypoperfusion (from M.I.) can cause global cerebral ischemia (H.I.E.) Key: Watershed infarct occur between zones of perfusion of Anterior, Middle and Posterior cerebral arteries. Hallmark of Watershed zone infarcts: Bilateral, wedge-shaped strips of necrosis over cerebral convexity, parallel and adjacent to longitudinal cerebral fissures. Note: permanent brain damage occurs after 4-5 minutes.

Rhinovirus (common cold) receptor?

ICAM (CD54)

45 y/o man brought to ER with new-onset seizures. Blank stare followed by Generalized tonic-clonic convulsions with tongue biting and urinary incontinence. Noncontrast head CT and brain MRI show Cavernous Hemangioma. What is he at greatest risk for?

INTRA-cerebral hemorrhage Key: Cavernous hemangiomas are vascular malformation within brain parenchyme--->can rupture and cause Intracerebral hemorrhages and seizures (brain tissue irritation)

What does +Straight Leg Raise Test for?

If pain occurs in the supine pt on Straight Leg Raise, it's a sign of Sciatic nerve root (L4-S3) irritation. Hallmark of Intervertebral disc herniation causing Sciatica Sciatics--->

Landmark for lumbar puncture?

Iliac crest (between L3 and L4) Key: Must do lumbar puncture for Meningococcal meningitis to analyze CSF. Spinal cord extends from Medulla to L1 vertebrae (adults) and L2-L3 in neonates/infants. L4 vertebrae sits right on top of Iliac crests, so go in between L3-L4, or L4-L5 (well below spinal cord termination point of L1).

13 y/o boy comes in after post-op Appendectomy. Several days later, he is having burning pain at the surgical scar site, radiating to suprapubic region. Loss of sensation over right suprapubic area. Normal Cremasteric reflex. What nerve was injured?

Iliohypogastric nerve Key: Iliohypogastric nerve provides sensation to: Suprapubic and Gluteal regions and also innervates Motor function to Anterloateral Abdominal wall. So, appendectomy can damage Iliohypogastric nerve and cause decreased sensation/burning pain at suprapubic region.

Cross section of brain of 54 y/o man who suffered from severe Tuberculous meningitis. Impaired function of what?

Impaired function of: Arachnoid granulations Key: Symmetrical enlargement of Ventricles in brain is hallmark of "Communicating Hydrocephalus" occurs secondary to dysfunction in Arachnoid granulations, due to a Meningitis infection (like T.B. meningitis)

Injury to obturater nerve?

Impaired thigh ADDuction, loss of medial thigh sensation.

Prader Willi Syndrome?

Imprinting disorder, caused by PATERNAL deletion of a part of Chromosome 15.

What is the striatum?

In H.D. you have bilateral atrophy of Caudate and Putamen (together form the Striatum) High Dopamine, Low Ach, Low GABA

How is M.G. different from LEMS or Botulism?

In M.G., the Ach concentration is unaffected (only receptors are attacked) In LEMS and Botulism, Ach conc is affected at synapse. Note: In MG, the decreased compound muscle action potential amplitude (summed action potentials of all muscle fibers in motor unit) on ECG is low with repeated excitation.

When does Communicating Hydrocephalus occur?

In Meningitis (due to destruction of arachnoid granulations),

How does the capsular polysaccharide of encapsulated bacteria help?

In P SHiNE SKiS, the capsule helps the bacteria resist phagocytosis. Capsular polysacchardides allow Survival in the host, but DO NOT correlate to morbidity and mortality.

Which enzyme is the rate-limiting step in the PPP, necessary for production of NADPH?

In PPP, G6PD catalyzes the rate limiting step. Needed for production of NADPH for proper regeneration of Glutathione and antioxidant effects. G6PD deficiency leads to normocytic hemolytic anemia.

When do "red neurons" develop?

In first 12-24 hrs after ischemic stroke, affected neurons shrink and become intesenly Eosinophilic (red neurons) with nuclear pyknosis

What happens to compound muscle action potential in M.G.?

In normal people, the threshold voltage must be exceeded (via neuromuscular Motor end plate potential), in order to allow for action potential propagation. In M.G., motor end plate potential is not reached, threshold isn't crossed, so action potential can't really occur. Note: In M.G., the "compound muscle action potential amplitude (aka ability of multiple action potentials occurring back to back) is decreased, especially with repeated excitation.

66 y/o man comes in with diplopia, worse at end of day, hard time chewing food. Started on low-dose Pyridostigmine, with improvement of S+S. P.E. shows "fatigable" asymmetric ptosis, and binocular horizontal diplopia. Infusion of Edrophonium helps. Best next step?

Increase Pyridostigmine dosage Key: Guy has MG. Tx him with long-acting AchE inhibitors (Pyridostigmine) helps S+S. However, the dosage may be too low if his S+S persist mildly. Can test by giving him the Edrophonium challlenge test (Tensilon), which is a short-acting AchE inhibitor. If S+S improve, then means pt needs increased dosage of the AchE inhibitor (increase dose of Pyridostigmine)

19 y/o with sustained traumatic brain injury during a MVA. GCS is 3, pt is comatose. Pupils are miotic, equal, and reactive to light. Pt is intubated and put on a mechanical ventilator. Noncontrast CT of brain shows bifrontal contusions and a basilar skull fracture. Repeat CT shows diffuse cerebral edema. The Ventilator Resp Rate (RR) is adjusted so that PaCO2 is 26-30 mmHg. Effect of this?

Increased cerebral vascular resistance Key: CO2 in general is a very potent vasodilator of cerebral vasculature. So, by decreasing the ventilator setting of PaCO2, it allows for cerebral vasoconstriction, which thereby will decrease the cerebral blood flow and decrease the ICP.

Pt with HIV complains of headache and fever. India ink of CSF shows spherical yeasts with thick capsules. Primary focus on infection?

India Ink would stain classic: Cryptococcus Neoformans Crytpococcus neoformans is in soil and random pigeon droppings... The yeast is transmitted via respiratory route to---> Lungs. The lung infection then disseminated to CNS (brain) and causes a headache, etc. in +HIV pts. Note: C. Neoformans has very thick polysacchardide capsule with antiphagocytic property. It's a fungus in soil and pigeon poop.

15 y/o high school athlete has severe left leg pain. Felt popping sensation while practicing gymnastics. Developed anterior leg pain after landing a jump. Swelling of left knee, with tenderness above patella. Unable to raise left leg against gravity. Imaging show complete tear of quadriceps tendon. Where to inject Lidocaine?

Inguinal crease Key: A femoral nerve block at inguinal crease will anesthetize the skin and muscles of anterior thigh (quads), femur, and knee.

Lab animals are given a toxin that inhibits protein Kinesin. What would this inhibit?

Inhibiting Kinesin would inhibit Secretory vesicles in nerve terminals Kinesin is a microtubule motor protein that functions in anterograde transport of intracellular vesicles and organelles toward the +end of microtubules. Kinesin will carry the vesicles and organelles toward synaptic nerve terminal. Toxin that inhibits Kinesin would inhibit anterograde transport of synaptic vesicles toward nerve terminal.

Man tried to commit suicide on prescription drugs. On P.E., he has rigidity, and +cannabis use. His mom and brother recently were started on Risperidone for Schizo. Elevated Cr and CK. Tx?

Inhibition of calcium ion release from SR of skeletal muscle Key: Pt has "Neuroleptic Malignant Syndrome" which occurs due to adverse effect of antipsychotics. S+S muscle rigidity, hyperthermia, altered sensorium. Tx: Immediately stop the antipsychotic, and give Dantrolene (muscle relaxant that inhibits release of Ca2+ from SR)

25 y/o man with uses crutches for broken R. tibia. Now has R. UEX weakness and numbness. Diminished wrist extension, with absent triceps reflex. Injury to what nerve?

Injury to Radial nerve Key: MSL = Rad forced trauma on Axilla from crutches can damage Radial n. Inhibits wrist extension--->causes Wrist drop (Saturday night palsy) and absent triceps reflex

What happens if the Hemisection in BSS occurs ABOVE T1.

Injury to T1 vertebrae would cause IPSILATERAL Horner's Syndrome (ptosis, myosis, anhydrosis,) due to damage to cervical SNS ganglion.

32 y/o woman with N/V episodes. Sensation of room spinning or tilting. Hard for her to walk without losing balance. +Romberg test. Dysfunction of what structure?

Inner ear Key: pt has classic vertigo, with sensation of room spinning and stuff. Due to inner ear problems with Vestibular system.

Woman with LEX weakness. Progresive bilateral leg weakness and hard time walking. Temp is 101, and she is confused, and has a "Morbilliform" rash on trunk and arms. Coarse hand tremor and flaccid paralysis of bilateral LEX. CSF on lumbar puncture shows viral RNA. What agent infected her?

Insect bite Woman has classic West Nile Virus +positive sense, ss Flavivirus transmitted by female mosquitos in summer. Causes encephalitis, meninigits, and/or flaccid paralysis.

28 y/o male has fever, muscle rigidity after surgery. Has muscle stiffness and cyanotic skin mottling. Tx?

Intracellular Calcium release in skeletal muscles Key: Pt has classic "Malignant Hyperthermia" after admin of inhaled anesthetic (Halothane gas) and/or Succinylcholine. Tx MH with: Dantrolene, which blocks Ryandodine receptors and prevents release of Calcium into cytoplasm of skeletal muscle fibers.

light microscopy of brain tissue shows neurons with: Shrunken nuclei No Nissle substance and Intensely Eosinophilic cytoplasm These findings point to?

Irreversible cell injury Key: A neuron responding to irreversible cell damage is a "Red Neuron" Red red neurons become evident within 12-24 hours after an injury. Includes shrinkage of cell body, (nuclear pyknosis), intense Eosinophilia of cytoplasm, and loss of Nissl substance. Followed by cell death of neuron. Remnants are phagocytosed by Microglia. Astrocytes proliferate at injury to form Glial scar

S+S of organophosphate poisoning?

Irreversibly inhibit AchE in both M+N receptors. Causes excess Ach in synaptic cleft. DUMBELS D: diarrhea, drooling, diaphoresis (sweating) U: Urination (stimulation of detrusor muscle) M: Miosis (pupillary sphincter constriction) B: Bronchospasms (increased bronchial smooth muscle tone) E: Emesis (N/V) L: Lacrimation S: Salivation KeyAF: All these DUMBELS effects are only due to Muscarinic receptor overactivation.

Jugular Foramen contains?

JFK= CN 9,10,11

Lesion to Jugular Foramen?

Jugular Foramen (9,10,11) Lesion causes dysphagia, hoarseness, loss of gag reflex (on side of dysfunction), and deviation of uvula TOWARD normal side (runs away from da problem)

67 y/o man has history of lung cancer. He has difficulty swallowing, hoarseness, loss of gag reflex on Left side, and Uvula deviates to Right side. His Left shoulder is drooped and strength is reduced during Left should shrug. Lesion to what structure?

Jugular foramen (JFK, 9, 10, 11) Lesion to Jugular Foramen affects CN 9, 10, 11 S+S: dyspahgia, hoarseness, loss of gag reflex on IPSILATERAL side of lesion, and deviation of uvula to OPPOSITE side of lesion (normal side). Also have shoulder drop due to CN 11 (SCM and Trapezius) dysfunction.

Palpable flank mass in kids 2-4 y/o

KEY: Wilms Tumor (Unilateral Kidney tumor)(Wilms tumor aka Nephroblastoma) occurs in kids 2-4 y/o. Presents as a palpable flank mass when bathing the child.

In a clinical experiment, it's determined that RMP is determined by high membrane permeability to a particular ion. When neurotransmitter stimulation begins, the ligand-gated ion channels open and increase the membrane permeability of a different ion. This causes a change in the RMP. This process triggers opening of Voltage-gated Ca2+ channels, which increases membrane permeability of a 3rd ion. What were the ions?

Key: RMP: Potassium Ligand-gated ion channel permeability: Sodium Voltage-gated ion channel permeability: Chloride Key: RMP is -70mV, which means more positive outside than in. So at RMP, there's more K+ inside Next, there's opening of ligand-gated ion channel in response to neurotransmitter binding. So this means membrane is now more permeability to an ion with +positive charge that rushes in (Na+) Finally, Volage-gated Ca2+ channels open due to the change in RMP. This causes the membrane to become more permeable to an ion with -Negative membrane potential (Chloride Cl-) and it now can go back down to -RMP.

Why would Glial hyperplasia occur in brain after irreversible neuronal cell damage?

Key: Astrocytes in brains are key for tissue repair in brain. In irreversible neuronal damage/neuronal cell death; Astrocytes seriously go to work and proliferate at site of injury. This Process is called "Reactive Gliosis/Reactive Astrocytosis" Key: The astrocytes work to replace the lost neurons and compensate for the lost volume so you don't end up with a big-ass hole in your brain!

44 y/o man brought to ER bc he has confusion, severe headache, and N/V. Nuchal rigidity, fever of 101. Gram stain of CSF would show what?

Key: Lancet-shaped, Gram+ cocci in pairs KEY AF: #1 MCC of MOPS (meningitis, otitis media, pneumonia, and sepsis) is always S. pneumo!!!! This patient has Bacterial meningitis. Hallmark of Bacterial meningitis: CSF with elevated neutrophils, low glucose, high protein. Second MCC of Meningitis: N. Gonorrhea (Gram negative diplococci) but usually only occurs in college dorms/military barracks/closer quarters.

MOA of N. meningitis virulence?

Key: N. meningitis are transmitted via respiratory droplets in Military Baracks/college dorms. It then colonized to nasopharynx, and attached to mucosal epithelial cells all via Pilli. Then it invades the vasculature via hematogenous spread and gains access to meninges by crossing BBB.

Staff members at college dorm noticed multiple students presenting with similar S+S: headache, sensitivity to light, N/V. Fever as high as 104F. Distinct rash appeared within day of onset of S+S. Developed septic shock with hypotension. What microbial component causes death in this condition?

Key: all the college kids have classic N. meningitis (distinct rash is petechial rash due to thrombocytopenia) The virulence factor of N. meningitis that causes death is the: Outer membrane Lipooligosaccharide (LOS) Key: The N. Meningococcal LOS causes the toxic effects observed in meningitis, causing the septic shock and death. Must screen blood levels of LOS to predict morbidity/mortality.

reactivation sores/blisters around mouth (HSV-1) herpes labialis (cold sores) requires what transport process?

Kinesin Kinesin is a microtubule-associated transport protein that is involved in Anterograde transport of stuff in cells. Reactivation of HSV requires anterograde transport of viral particles from neuronal cell bodies in DRG of spinal cord to skin/oral mucosa. Note: In primary HSV-1 infection, Dyenin allows retrograde transport of virus from skin backwards to Spinal cord (DRG) to remain latent.

Which sphingolipid disease causes accumulation of Galactocerebroside?

Krabbe's Disease Causes accumulation of Galactocerebroside. Like Tay Sachs, Krabbe's disease also causes progressive neurodegeneration. Hallmark of Krabbe's: neurodegeneration, neuropathy, and Optic atrophy.

Difference between LMN and UMN?

LMN (Anterior horn) UMN (Lateral CST). Ex: So if it says "Ipsilateral Spastic paralysis BELOW level of lesion" then you know it's talking about the Lateral CST of UMN injury Ex: So if it says Ipsilateral flaccid paralysis AT level of lesion, then you know it's talking about the Anterior Horn of the LMN.

Where are multiple lacunar infarcts seen?

Lacunar Infarcts occur in: Vascular Dementia. Has a "step-wise" quicker decline with FOCAL neurological deficits.

Cause of Hemiballism?

Lacunar infarct (due to HTN) in Subthalamic nucleus causes contralteral Hemiballism.

What was ruptured?

Lady has a Subdural Hematoma due to: rupture of Cortical Bridging veins in young pts: SDH occurs with fall/MVA and causes "gradual onset" headache and confusion In elderly: SDH can occur with even minor trauma SDH causes blood between Dura and Arachnoid. Blood comes from bridging cortical veins. "Crecsent-shaped mass" on CT

Other anticonvulsants that could cause SJS/TEN?

Lamotrigine, Carbamazepine, Phenobarbital. Phenytoin are all anticonvulsants that can all causes SJS/TEN.

Radial nerve MSL

Largest nerve of Brachial plexus injured during injury to Midshaft of humerus (radial groove on humerus) injury during crutches (sprial groove in Axilla) Lateral epicondylitis (tennis elbow) Injury of Radial nerve: Wrist drop, weak wrist extension, sensory loss over Posterior arm and forearm Function of Radial nerve: 1. Extension of arm, wrist, fingers 2. Forearm Suppination 3. ABduction of thumb (in anatomical position)

20 y/o man has Right arm weakness, numbness. He is a Professional Baseball pitcher. Hard time using R. arm, or lifting stuff with it. Diminished Right elbow flexion and absent Biceps reflex. Sensory loss over where?

Lateral forarm Key: The Musculocutaneous nerve innervates the major forearm flexors (Biceps Brachii, Brachialis) as well as the Coracobrachialis (flexes and ADDucts arm) KEY: Musculocutaneous nerve provides sensory innervation to LATERAL forearm. Derived from Brachial plexus (C5-C7) Injury occurs with trauma/strenuous UEX workouts like pitching, shoulder dislocation/

Zapping Lateral nuclei of Hypothalamus does what?

Lateral nuclei mediates hunger (via Ghrelin) Zapping it causes anorexia "Zap Lateral nuclei causes you to shrink Laterally"

What does Ach overconc. at Nicotinic receptors do?

Leads to muscle cramps, and paralysis due to constant contraction.

55 y/o man with poorly controlled HTN, and T2 D.M. Blurry vision, shadow over left eye. "Flame-shaped hemorrhage" in Left Temporal Hemiretina. Transmission of visual input through what structure is impeded?

Left Lateral Geniculate Body Key: Pt has classic HTN flame-shaped hemorrhage in Left Temporal hemiretina. Left Retina receives visual info from Left eye visual field. Vision info is transmitted from 1. Left optic nerve--->2. L. Lateral optic chiasm---3. Left Optic tract--->4. Left Lateral Geniculate nucleus (in Thalamus)---> Finally to Left sided primary visual cortex for actual visual processing in back of head.

23 y/o woman with recent onset of Diplopia and Ataxia. A year ago, had numbness and paresthesia in left leg. Dr orders an MRI of brain, which shows axonal demyelination. Dx: is M.S. What neuronal properties is decreased as a DIRECT result of demyelination?

Length constant Key: Length constant is a measure of how far along an axon an electrical impulse can propagate. (Ex: low length constant reduces the distance an impulse can travel) Myelination normally increases the length constant and decreases the time constant (both which improve axonal conduction speed) So, demyelination in M.S. impairs stimulus transmission.

Pt with Lesch Nyann Syndrome would have an increase in which enzyme?

Lesch Nyann Syndrome is an X-linked recesive dx where there is a defect in HGPRT (Hypoxanthine-Guanine Phosphoribosyltransferase). Defect in "Purine Salvage Pathway" Lack of HGPRT means lack of conversion of Guanine into GMP and Inosine into IMP. Causes conversion to Inosine--->Hypoxathine--->Xanthine---->Uric Acid So: Patient MUST have a compensatory Increase in Purine Salvage pathway to replace the lost bases by increasing PRPP. (Phosphoribosyl pyrophosphate aminotransferase)

Lesion in Wernicke's Aphasia?

Lesion is in Superior Temporal Gyrus (Left Dominant Temporal Lobe) S+S: "Word Salad" Wernicke's (Receptive) Aphasia leads to loss of understanding of language. Lack of comprehension, so can't write, don't know there's something wrong with them, can't read, can't understand, can't repeat.

64 y/o male smoker, has nagging right shoulder pain, that radiates to ipsilateral arm. Also has weakness in RUX. Partial right-sided ptosis with fully intact EOM. Pupils are assymetric in dim light but both are reactive to light. Autonomic dysfunction results from lesion to what?

Lesion to Autonomic ganglia Pt is a 65 y/o male smoker= classic Pancoast tumor (from non-small cell lung cancer like Squamous Cell Carcinoma, or Adenocarcinoma) arises near the: Superior Sulcus (groove near Subclavian artery)! S+S: Ipsilateral shoulder pain, upper limb paresthesia, areflexia/arm weakness (due to involvement of brachial plexus). Horner's syndrome (ipsilateral ptsosis, miosis, anhydrosis) due to involvement of cervical sympathetic ganglia.

Lateral Medullary Syndrome?

Lesion to Lateral medulla causes Lateral Medullary Syndrome (Wallenberg): S+S: contralateral loss of pain and temperature, along with ipsilateral paralysis of CN 5, 9, 10, 11.

Lesion to MLF?

Lesion to MLF causes "Internuclear ophthalmoplegia" associated with MS. Ex: When you try to look to the Right, the MLF causes Right eye LR (CN6) and Left eye MR (CN3) to work. Lesion to Right MLF; when you try to look to the Left, impaired Right eye CN3 (MR) so can't ADDUCT right eye. Impaired Left eye LR CN6, so can't ABduct, which causes left sided nystagmus.

Lesion to MLF?

Lesion to MLF occurs in M.S. S+S: Left INO: So when You try to Look to the Right: The Left eye has problem ADDUcting, so it basically looks straight ahead. The Right eye therefore has ABduction to the Right, with Nystagmus as well (rapid involuntary eye movements)

MOA of combination of Levodopa and Carbidopa?

Levodopa (L-Dopa) is inhibited from being converted to DA in periphery (to inhibit S+S of N/V) via Carbidopa. Carbidopa inhibits Dopa-Decarboxylase in periphery (but does not cross BBB)

What is a lacunar infarct?

Lipohyalinosis with small vessel occlusion: lacunar infarcts result from small vessel occlusion (due to lipohyalinosis and microatheroma formation). in penetrating vessels supplying deep brain structures. Risk factors: Uncontrolled HTN/DM. Can lead to liquification necrosis in CNS due to small vessel occlusion.

73 y/o man brought to ER after right sided hemiparesis. Difficulty speaking. Has history of A-Fib. He has irreversible cell injury responsible for his current S+S, due to which histological change?

Liquefactive necrosis Key: Necrosis results in IRREVERSIBLE cell changes. Irreversible brain injury causes: Liquefactive necrosis. Key: The infarcted CNS is replaced by a "cystic astroglial scar"

P.D. loss of?

Look at other deck

Labs for Down's Syndrome?

Low AFP, and a normal AChE, high Beta-hCG

5 month old boy has developmental delay. Immigrated from South America. Can't roll from front to back or back to front, doesn't recognize parents. Labs show impaired: Tetrahydrobiopterin synthesis. What's deficient?

Low levels of 5-HT (Serotonin) Key: Tetrahydrobiopteryn is a cofactor for synthesis of: Tyrosine, DA, Dopamine. Key Key: PKU= results due to lack of BH4 (THB). S+S: mentally retarded bc of lack of Serotonin and also high levels of Phenylalanine. Diet of PKU pt: Low phenylalanine and take some BH4 supplements.

46 y/o man brought in by wife. Says he acts strange for past 6 months. Doesn't care about his family, sometimes is aggressive, and has jerky movements. What is deficient?

Low levels of GABA Key: Huntington's Dx is an A.D. condition. Causes high Dopamine, Low Ach, low GABA. Huntington's dementia, chorea, depression. Key: Loss of GABA in Striatum due to increased # of trinucletotide repeats (CAG) on Chromosome 4 (HUNT)

Defects in DNA mismatch repair enzymes cause?

Lynch Syndrome (HNPCC) occurs due to defect in DNA mismatch repair enzymes.

32 y/o female Caucasian, compains of Left eye pain. Periodic dimming of vision in Left eye, especially after a hot shower or intense work out. Episodic numbness and tingling in Left arm sometimes as well. Dx?

M.S. Key: This is classic M.S. in a middle aged white girl! S+S: Optic Neuritis, INO, n, t, p., bladder and bowel dysfunction. S+S worsen with heat exposure (hot shower, sweat from work outs)

What pathway do both the M1 and M3 receptors use?

M1 and M3 Muscarinic receptors use the IP3 (Inositol triphosphate) pathway in order to increase intracellular Ca2+ concentration. Note: M2-receptors act via cAMP pathway, resulting in decreased intracellular cAMP concentrations.

Action of M2-receptors?

M2-receptors are in the cardiac muscle. Activation will decrease G-protein, decrease cAMP, open up K channels and slows depolarization. Causes decreased Inotropy (Force) and decreased Chronotropy (HR)

What is MAC?

MAC measures potency of inhaled anesthetic MAC is the concentration of the anesthetic in the alveoli, that renders 50% of pts unresponsive to painful stimuli (ED50) So: Lower MAC, higher potency Higher MAC, lower potency

MCA stroke?

MCA stroke causes CONTRALATERAL motor and sensory defecits, more so in UEX. Causes Homonoymous hemianopia with macular INVOLVEMENT.

Epidural Hematoma causes rupture of?

MMA causes Epidural hematoma Hallmark: "Lucid interval" followed by LOC CT shows: Biconvex shape

What test is given to assess attention/concentration?

MME (mini mental state exam)= used to assess for cognitive impairment

55 y/o man has headaches, throbbing, N/V. Worsens when he coughs or bears down while pooping. History of migraines without aura. MRI of brain shows cystic mass:

MRI is showing a neoplasm in Cerebellum. Key: Cerebellum is largest structure in posterior fossa, and has 2 hemispheres with the Vermis in the midline. Function: Cerebellar hemispheres allow motor planning and coordination on IPSILATERAL side via connection with Lateral (Descending) CST (motor movement) Note: Bc pt has a cerebellar tumor in Right Cerebellum (switch R/L!!), it's gonna cause 1. Right sided Dysdiadochokinesia (impaired rapid movements), 2. Right limb dymetria (under/overshoot target) 3. intention tremor (occurs with intentional targeted movement like picking up a fork)

Middle Cerebral Artery on Circle of Willis

Make sure to Flip Left and Right!!

MOA of MH?

Malignant hyperthermia occurs due to defect in Ryanodine receptors in SR. Ryandodine receptors on SR of skeletal muscle is a type of Calcium channel. Supposed to release small amount of Calcium in cytoplasm of muscles to contract. Abnormal defect in RNR after inhaled anesthesia or Succinylcholine causes RNR receptors to release wayyyyy too much Ca2+, which causes excess consumption of ATP. This generates a ton of heat, and causes muscle damage. Rhabdomylosis ensues, releasing a ton of: K+, Myoglobin, and Creatine kinase into circulation. S+S: fever, muscle rigidity, tachycardia, HTN, hyperkalemia, myoglobinuria, etc.

Man with Schizophrenia has sudden onset pain and stiffness on side of neck. Head it tilted to one side and he cannot straighten it without pain. Antagonism to what receptor causes these S+S?

Man with Schizo is treated with med inhibiting D2 receptors, and is now having spasmodic torticollis. Inhibition of D2 receptors causes Acute extrapyramidal S+S (dystonia, akathisia, Parkinsonism) Ex of meds that do this: High potency antipsychotics like Haloperidol, Fluphenazine= both will strongly block D2 receptors.

Man has painful tongue sore, myalgias, and arthralgias. Had unprotected sex with a total stranger 1 month ago (smh...) The ulcer on the "median sulcus of the tongue" is 2 cm anterior to foramen cecum. Pain from ulcer carried by what nerve?

Mandibular division of CN 5 Key: This dumb man has now contracted HIV (presents as rash, fever, lymphadenopathy, painful oral ulcers on tongue) Since the ulcer is 2cm anterior to foramen cecum, it's in Anterior 2/3rds of tongue. It's innervated by V3 (Mandibular branch)

26 y/o woman with muffled hearing in left ear. Sensation of jaw clicking when chewing food. Left sided facial pain for few years. Can't open mouth fully. Develops pain upon passive motion of jaw. Left medial pterygoid muscle dysfunction is suspected. What CN is involved?

Mandibular division of Trigeminal CN 5 Pt has TMJ (temporomandibular disorder). Problems with muscles of mastication (CNV3 (chew) Key: (Mandibular) V3 branch of Trigeminal nerve supplied middle ear and muscles of Mastication (Masseter, Medial/Lateral Pterygoid, PlatysMa)

If Lateral forearm is innervated by Musculocutaneous nerve, what about the medial forearm?

Medial aspect of forearm is innervated by Medial cutaneous nerve

Job of Medial optic chiasm?

Medial optic chiasm (unlike Lateral optic chiasm) actually crosses its nerve fibers so that it can transmit visual info to bilateral nasal hemiretinas.

Function of Posterior Hypothalamus?

Mediates heat conservation; destruction leads to Hypothermia.

Most common Cerebellar tumor in kids?

Medulloblastoma S+S: gait and limb ataxia, intention tremor, Nystagmus

What common brain tumor in kids arises from Cerebellum?

Medulloblastoma is the 2nd Most common brain tumor in kids. Arises from Cerebellum. (#1MC brain tumor in Kids is Craniopharyngioma) Medulloblastoma causes ataxia

Where do Medulloblastomas occur (kid tumor)

Medulloblastomas are Posterior Fossa tumors (Cerebellum) S+S: Increased ICP, Cerebellar dysfunction (dizzy, gait ataxia, nystagmus)

Most common malignant brain tumor in kids?

Medulloblastomas are malignant brain tumors in KIDS hallmark: "really really small blue cells" with "Homer-Wright" rosettes

What does she have?

Meningioma Key: Meningiomas are well-circumscribed, slow growing, benign tumors. Hallmark: "whorled patterns" of growth that form "nests" which can calcify into round eosinophilic structures (psammoma bodies) S+S of overgrowth of Meningioma: seizures due to compression of cerebral cortex. Headahces, N/V due to increased ICP.

Which brain tumor arises from the Arachnoid matter, and has psammoma bodies (whorled pattern of calcification)?

Meningioma Slow-growing, arises from Arachnoid matter Cells are in Whorled pattern (Psammoma body calcifications)

What are the benign, well-circumscribed neoplasms of adults?

Meningiomas are benign, well-circumscribed neoplasms of adults that arise from Arachnoid cells.

The cells marked most intensely in the lipid stain represent which cell type?

Microglia Pt had stroke, resulting in ischemic infarct (blood flow to area of brain in interrupted due to thrombus/embolus) key: changes in brain appear 12 hours after onset of ischemic infarct. Neurons will be irreversibly damages during first 48 hours after injury (red neurons). First neutrophils move to that area, followed by a ton of microglia 3-5 days after ischemia. As neurons disintegrate, their fragments are phagocytosed by Microglia. Pic shows Microglia filled with necrotic debris about a week after ischemic infarct.

Function of Microglia?

Microglia are the macrophages of the CNS Function: Phagocytosis.

44 y/o +HIV man has progressive cognitive decline. Worsening memory, poor tx compliance. 2 weeks later, dies of severe pneumonia and respiratory failure. Histo of brain biopsy would show what?

Microglial nodules +HIV dementia should definitely be suspected in AIDS pts with progressive cognitive decline. Histo would show microglial nodules, with a ton of activated macrophages/microglial cells around small areas of necrosis.

What do microvilli contain?

Microvilli contain actin thin filaments (not microtubules)

Compression of entire brainstem can cause what?

Midbrain, pons, medulle compression can cause "Cushing's Triad" HTN, Bradycardia, Resp. depression

Function of Cerebellar Vermis (at midline) associated with Medial aspect of Cerebellum?

Midline Cerebellar vermis allows for axial/truncal posture and coordination, via connection with Medial (descending) CST Lesion to medial Cerebellum causes: 1. Truncal ataxia (wide-based gait, unsteady gait) 2. Vertigo 3. Nystagmus

What can be used to treat migraines?

Migraine headaches cause unilateral, throbbing headache, with aura, N/V, photophobia. Tx: Sumatriptan

MOA of Triptans?

Migraines occur most commonly in younger women. S+S: phono/photophobia, aura sometimes, N/V, throbbing unilateral headache Trigeminal nerve is affected, causing the severe pain (innervates meninges). Also causes release of Vasoactive Neuropeptides (Substance P), which causes neurogenic inflammation/vasodilation/pain. Triptans (Sumitriptan) are 5-HT agonists. Used for acute onset migraines (during episode). MOA: inhibits release of vasoactive peptides like Substance P. Allows vasoconstriction, blocking pain pathways in brainstem. Side Effect: Triptans can cause elevated BP and cardiac events, so avoid in pts with Cardiac or Cerebrovascular Dx.

Naloxone (Narcan) blous increases the RR of an opioid overdose. S+S of an opioid overdose?

Miosis, bradycardia, hypotension, decrease RR, decreased bowel sounds.

What is Heteroplasmy?

Mixture of two types of genetic material (some mitochondria with damaged DNA, other mitochondria with normal DNA), causes clinical variability of S+S in mitochondrial diseases.

Absence seizure

Most commonly occurs in children Can occur many times in a day Brief episodes (~10 seconds) of unresponsiveness blank stare No postictal deficits and automatisms

Which inhaled gas anesthetic can cause Hepatotoxicity?

Most inhaled gases are Cleared via exhalation into air. (b/c liver metabolism is slow) Halothane, however, is metabolized by Liver the most, and produces reactive intermediates that can cause severe hepatoxicity.

Which lysosomal storage diseases cause buildup of of GAG (glycosoaminoglycans)

Mucopolysaccharide dx: Hurler's or Hunter's Dx GAG accumulation causes coarse facial features, neurocognitive decline, skeletal probs, etc.

Muscle fiber inflammation and necrosis is seen in?

Muscle fiber inflammation and necrosis occurs in Myositis (Polymyositis, Dermatomyositis,)

6 month old girl brought to clinic, mom says she can't latch on to nipple for breast feeding. She can roll over when prone, but cannot roll when supine or sit up. She has a huge head and full anterior fontanelle. CT shows dilation of lateral ventricles. Future complication if left untreated?

Muscle hypertonicity Kid has classic Hydrocephalus with Macrocephaly and poor feeding. Brain CT will show enlarged lateral ventricles. Untreated hydrocephalus causes: hypertonicity and spasticity, bc you're gonna stretch the periventricular pyramidal tracts, and lead to developmental delays and seizures!

What CN innervates larynx?

Muscles of Larynx are innervated by Vagus nerve (specifically the recurrent laryngeal nerve branch of CN10)

What sensation are the Inferior Colliculi and Medial Geniculate bodies associated with?

Music

Why is Apolipoprotein E bad?

Mutation in Apo E impairs clearance of Beta Amyloid from brain. Causes late-onset A.D.

21 y/o white male has gait probs. Probs with releasing doorknobs. P.E. shows cataracts, frontal baldness, gonadal atrophy. Muscle atrophy of Type 1 fibers. Dx?

Myotonic dystrophy (affects Type 1 Fibers) Myotonic dystrophy is an A.D. dx. Increased number of trinucleotide repeats on myotonic protein gene. S+S: sustained muscle contraction (myotonia), weakness, atrophy of gonads, cataracts (in all pts). Can also cause balding. M.D. is the second MCC of inherited muscle dx. (A.D. trinucleotide repeat of CTG)

MOA of NMDA-receptor

N-methyl-D-aspartate receptors are inhibited by Ketamine, which will then block activity of Glutamate (excitatory n.s.) Blocking NMDA-glutamate receptors allows anesthesia, sedation, memory loss.

How is LOS different from LPS?

N. meningitis has an outer membrane LOS (Lipooligosaccharide) which is basically the same as LPS in Gram- bacteria, but LOS lacks the repeating O antigen that LPS has. LOS acts as an Endotoxin, causing the toxic effects of N. meningitis infection, eventually causing septic shock and death. Note: N. meningitis is "G- diplococci that's kidney bean shaped"

S+S of opioid withdrawal?

N/V, myalgias, dilated pupils, diaphoresis, tachycardia.

NADH and NADPH are both derived from what Vitamin?

NADPH and NADH both come from Niacin (Vit B3)

Inheritance pattern of NF1?

NF-1 (von Reckinghausen's Dx) is an A.D. disorder due to mutation of Chromosome 17. S+S: cafe-au-lait, neurofibromas, Lisch nodules Note: NF-1 has high "penetrance," meaning most people that inherit it will exhibit the phenotype. Key AF: NF-1 is a MEN-1 disease, so it can occur along with PCC. Can also occur with meningiomas, astrocytomas, gliomas (brain tumors along with NF-1)

24 y/o woman complains of tinnitus, vertigo, and hearing loss. Mutation of what gene?

NF2 on Chromosome 22 (A.D.) (she has Bilateral Acoustic Neuroma)

MOA of H.D.

NMDA (N-methyl-D-Aspartate) receptors bind Glutamate and cause neuronal cell death (NMDA toxicity)

59 y/o female has diplopia, long-standing Diabetes, poor glycemic control. Right sided ptosis, with Right pupil in Inferolateral position. Caused by what?

Nerve Ischemia Key: The Diabetic retinopathy often involves: CN3 palsy (down and out dilated pupil) The buildup of Sorbitol junk inhibits blood flow and causes Nerve ischemia.

43 y/o woman can't sleep bc of tingling in 1st three digits on both hands. Also has chronic R.F. due to uncontrolled HTN, and receives Dialysis. Bilateral diminished sensation in index, middle and radial half of ring finger. MOA of pt's S+S?

Nerve compression within anatomic compartment Key: This pt's history of HTN, R.F. and dialysis actually is a clue that she has Beta2-Microglobulin (dialysis-associated amyloidosis) depositing in the carpel tunnel. Key: This pt has classic Carpel Tunnel Syndrome. Compression of median nerve (supplying middle of palmar wrist) causes weakness in Thumb ABduction/Opposition, Thenar muscle atrophy, and +Tinnel or +Phalen Test.

22 y/o man with dull headaches that awaken him from sleep. Several 3-to-5 cm flat, pigmented spots on his trunk. Multiple subcentimeter, soft, fleshy cutaneous tumors on trunk and neck. The cells forming these skin tumors originate from what?

Neural crest Key: Pt has NF1 (von Recklinghausen Dx) an A.D. disorder due to mutation in NF1 tumor suppressor gene. Pts develop numerous pigmented spots and cutaneous neurofibromas made of SCWHAAN cells from neural crest (PNS MES)

2 y/o boy with spontaneous bursts of non-rhythmic conjugate eye movements. Has hypotonia and myoclonic jerks. P.E. reveals an abdominal mass. Dx?

Neuroblastoma is due to lots of copies of N-myc gene. Key: In kids with Neuroblastoma, they develop non-rhythmic conjugate eye movements with myoclonic jerks (aka "Opsoclonus Myoclonus Syndrome") Neuroblastoma is the #1MCC of childhood tumor. KEY: It is NOT located in the brain, but rather in the Adrenal Medulla (the palpated "abdominal mass")

Day 24-72 hrs s/p Ischemic Stroke?

Neutrophilic infiltrate a shit ton!

When do Neutrophils migrate to area of ischemia?

Neutrophils are the first to migrate to ischemic area within 24-48 hours after interruption of blood supply. Then Microglia (CNS macrophages) within 3-5 days, which phagocytose necrotic debris Followed by Astrocytes (2 weeks) form Glial scar

Rabies receptor?

Nicotinic Ach receptor

Nondepol blocker vs Depol blocker?

Nondepolarizing blocker inhibits depolarization by inhibiting Ach from bidning to Ach receptor Depolarizing blocker inhibits break up of depol, causing constant contraction.

Why is MMA elevated in Vit12 deficiency?

Normally, MMA is a product of fatty acid oxidation MMA is converted to Succinyl-CoA via Methylmalonyl-CoA mutase (which uses Vit B12 as a cofactor) So, lack of Cobalamine, leads to buildup of MMA.

MOA of A.R. Methylmalonic acidemia?

Normally, Methylmalonic CoA forms Succinyl CoA through the help of Vit B12. Succinyl CoA then enters the TCA. Key: Mutation in Methylmalonyl-CoA mutase causes buildup of Methylmalonic Acid and Propionic Acid. This causes Metabolic Acidosis. Hypoglycemia also occurs due to increased use up of Glucose. Eventually causes Ketone bodies to form. Urea Cycle becomes inhibited as well, forming Hyperammonia. Net Result of defect in Methylmalonyl-CoA mutase: Increased A.G. Metabolic Acidosis, Hypoglycemia, Ketone Bodies, and Hyperammonia. Urine shows: Elevated MMA and Propionic Acid.

Which TCA is best for comorbid depression and anxiety?

Nortriptyline is a TCA used for neuropathic pain, migraines, especially in patients with both depression and anxiety.

42 caucasian male brought to ER by police. He keeps losing his temper and threatened his wife with a gun a few times. Wife says he recently grimaces involuntarily,, and his extremities move "without control." Neuron damage where?

Nucleus caudatus Key: Pt has A.D. Huntington's Dx (100% penetrance) Progressive depression, apathy, aggression, dementia, chorea. Loss of neurons in Caudate and Putamen Labs: high DA, low Ach, low GABA, low Substance P in Cuadate and Putamen of Striatum

Obturater nerve

Obturator canal contains obturater artery, vein, and nerve. Travels from pelvis to thigh, and innervates ADDuctor muscles of thigh, and also skin in medial thigh.

Obturator nerve?

Obturator nerve- ADDuction of thigh. innervates skin on medial thigh.

Cause of Lateral Pontine Syndrome?

Occlusion of AICA (Anterior Inferior Cerebellar Artery) causes: Lateral Pontine Myelinolisis S+S: of Right sided AICA stroke? 1. Right sided loss of pain and temp in face 2. Right sided facial weakness 3. Right sided loss of hearing 4. Left sided loss of pain and temp in Trunk and Extremities 5. Overall Cerebellar dysfunction causing ataxia

Stroke to MCA?

Occlusion of MCA would affect motor control of hand (gripping), face/mouth (whistling), and throat (swallowing). Key: MCA stroke can cause Broca's aphasia (damage of frontal lobe), hemispace spatial neglect of contralateral side (damage of nondominant parietal lobe), contralateral homonymous hemianopia.

Where are Oligodendrocytes derived from?

Oligodendrocytes are part of CNS. Derived from Neuroectoderm (like Astrocytes!) Function: Oligos form myelin in CNS yo!

Slow growing tumors of adults that involve white matter of cerebral hemispheres. Usually appear as well circumscribed gray mass with calcification.

Oligodendroglioma

What tumors look like a "fried egg"

Oligodendrogliomas involve white matter of cerebral hemispheres and look like a "fried egg"

Which anesthetic acts to block pain by blocking voltage-dependent Na+ channels?

Only local anesthetics (Lidocaine) can reduce pain transmission by blocking Voltage-dependent Na+ channels. This in turn decreases Na+ influx (depolarization) and inhibits the action potential propagation. Only works for minor surgical procedures.

44 y/o woman with Breast cancer, and chronic LBP. She takes Acetominophen and NSAIDs, but pain persists. Pt started on oral Morphine therapy. MOA of Morphine on spinal cord neurons?

Oral Morphine will increase Potassium (K+) efflux out of cells. Key: Opiate analgesics reduce pain by binding to the Mu-opiod receptors. This inhibits synaptic activity in CNS. Activation of Mu-opiod receptors in spinal cord neurons, causes closure of Voltage-gated Ca2+ channels, inhibiting Ca2+ influx. This in turn reduces excitatory neurotransmitter release, which decreases pain. Binding of the Mu-receptors to the postsynaptic membranes in spinal cord neurons also causes opening (efflux) of Potassium (K+) channels--->membrane hyperpolarization. This also reduces pain.

Organophosphate poisoning S+S?

Organophosphates poisoning causes: Direct inhibition of AchE Used in Insecticides. Organophosphates causes 1. a classic "Depolarizing" blockade (muscle weakness, fasciculations, and paralysis) 2. CNS effects like (lethargy and seizures) 3. Muscarinic "overstimulation" (miosis, bradycardia, lacrimation, salivation)

Where does P.D. hit?

P.D. causes degeneration of: Nigrostriatal pathway in P.D. is degenerated. Causes excess excitation of G.P. and Subthalamic Nucleus in Basal Ganglia. In turn, this causes excess inhibition of Thalamus, which causes hallmark TRAP. High-frequency Deep Brain Stimulation of S.N. and G.P can help inhibit the excess firing of these areas. This would in turn improve Thalamus activity, decreasing S+S of TRAP.

70 y/o Right handed woman comes to ER with Left-sided visual loss. Has history of HTN, A-Fib, and hyperlipidemia. Left homonymous hemianopia with macular sparing. Decreased sensation over entire left side of body. What cerebral artery is compromised?

PCA Key: PCA infarct on Right side would cause Left sided homonymous hemianopia, with macular sparing.

Occlusion where?

PCA Left sided homonymous hemianopia with macular sparing results from Right sided PCA occlusion.

Which stroke causes Contralateral Hemianopia, with macular sparing due to infarction of Visual Cortex?

PCA (Posterior Cerebral Artery) stroke causes: Contralateral homonymous hemianopsia, with Macular sparing. (bc PCA will compress down on visual cortex)

Occlusion of PICA?

PICA stroke causes the classic "Lateral Medullary (Wallenberg) Syndrome" Causes: Contralateral loss of pain and temp sensation Ipsilateral loss of CN 5, 8, 9, 10, 11 and Horner's Syndrome

Progressive Multifocal Leukoencephalopathy?

PML is caused by JC virus JC virus is a: ds DNA virus of Polyomavirus JC virus infects HIV pts, lymphoma/leukemia pts.

What is Progressive Multifocal Leukoencephalopathy?

PML occurs in immunocomp pts (HIV, Lymphoma, Leukemia) Cause of PML: JC virus infects CNS oligodendrocytes, causing demyelination. Causes loss of coordination and weakness.

Atonic Seizure (Death Drop)

Patients can present with "drop" seizures can be mistaken as fainting

Myoclonic Seizures

Patients can present with quick and repetitive jerks

Tonic Seizure

Patients can present with stiffening

4 y/o kid brought to ER with severe vomiting. He ate small brown mushrooms. He is in ICU bc he is so somnolent. The main poison in the mushroom stimulates Muscarinic receptors. Direct effect of mushroom?

Peripheral vasodilation Key: Over-Activation of Muscarininc receptors by Ach or Cholingergic agonists causes intense peripheral vasodilation and vascular smooth muscle relaxation (hypotension). Key: Muscarine is a toxin found in mushrooms that acts as an Muscarinic agonist, causing excess PNS activity. M3 receptor increases level of NO in endothelial cells, which then activates gyanylate cyclase, increases cGMP, increases MLCK, causing smooth muscle relaxation/vasodilation. Excess vasodilation causes "somnolence" due to improper brain perfusion to cerebrum.

Where are Peroxisomes most abundant?

Peroxisomes are cytoplasmic organelles containing oxidative enzymes, most abundant in Liver and kidney where detox of ingested and environment material occurs. In Liver, peroxisomes play role in breakdown of fatty acids.

65 y/o man comes to ER with slurred speech and RUX and RLX weakness. Woke up with these S+S. He has history of HTN, T2DM, Hyperlipidemia. Pt has 40 pack yr history. Brain MRI shows infarct in left internal capsule. If he comes back 6 months later, what would you observe in the affected Pyramidal tracts?

Persistent myelin debris Key: stem is just saying man had a stroke, and even if he comes back 6 months later, he would still have the persistent demyleination debris, bc CNS is not able to regenerate axonal damage. He WILL have a dense glial scar via Astrocytes doe. Key: Macrophages/microglia of CNS are unable to clear myelin debris after Wallerian degeneration, so can't regenerate. PNS: Macrophages/microglia can clear debris, can regenerate.

4th and 6th pharyngeal arches?

Pharynx, Cricoid, Thyroid cartilages, Larynx (CN10) Defect: hoarseness, esophageal dysmotility,

MOA of Phenobarbital?

Phenobarbital potenitates GABA activity, to decrease or cease seizures.

Tx of Generalized Tonic-clonic seizures?

Phenobarbital treats GTCS.

Man with history of mental illness and seizure disorders comes to office. Has yucky teeth, unhealthy gums, with teeth loss. Cause?

Phenytoin #1 Side effect: gingival hyperplasia Key: Phenytoin allows increased expression pf PDGF, which for some reason cause gingival macrophages to stimulate gingival cell proliferation. Phenytoin toxicity can also cause ataxia and nystagmus.

MOA of Phenytoin?

Phenytoin inhibits high-frequency action potentials by actually blocking Na+ channels. Good for both Generalized tonic-clonic seizures and status epilepticus.

78 y/o man undergoes bronchoscopy, and is premedicated with IM Atropine. Becomes acutely restless, disoriented, and combative. Widely dilated pupils, non-reactive to light. EKG shows sinus tachy. What agent will reverse ALL S+S?

Physiostigmine Key: Physiostigmine "Phixes" Atropine overdose in CNS and PNS. Atropine was given before bronchoscopy to decrease PNS mucus secretions and clear airway, and bronchodilate. However, this pt was given too much, and has Atropine toxicity (mydriasis, cyclopegia, bronchodilation, tachycardia, constipation, pee retention, hyperthermia, flushed skin, etc)

50 y/o man brought to ER with blurred vision while cutting trees in his garden. Temp is 102, flushed skin and dry oral mucosa. Pupils are dilated and non-reactive to light. What drug can reverse his condition?

Physiostigmine Pt has classic Atropine (Jimson weed/Belladona) poisoning: "Blind as a bat, mad as a hatter, red as a beet, hot as a hare, dry as a bone." Atropine is a reversible M3-antagonist that inhibits PNS effects. Effects can be reversed by Physiostigmine (an indirect Cholinergic agonist that inhibits AChE and increases Ach)

Difference between Physiostigmine and Neostigmine?

Physiostigmine is a (Tertiary amine) that reverses both CNS and PNS severe Atropine toxicity. It does so by inhibiting AchE AchE inhibitors like Neostigmine/Edrophonium are Quaternary amines, that cannot penetrate CNS.

MOA of Pilocarpine?

Pilocarpine is a nonselective M3-receptor AGonist ("Drooling on your pilo") would increase PNS effects.

14 y/o boy with N/V, headache, ataxia, vision probs. MRI shows cystic tumors in Cerebellum. Biopsy shows well-diff neoplasm, with "hair-like" glial processes. +Rosenthal fibers. Dx?

Pilocytic Astrocytoma Key: CYSTIC tumor in Cerebellum of kids is a Pilocytic Astrocytoma Biopsy will show "hair-like" glial projections. +Rosenthal fibers Key AF: Both Medulloblastoma and P.A can occur in Cerebellum in kids, but MRI is needed to differentiate bc Medulloblastoma is stricly SOLID, while P.A. is CYSTIC Also, P.A.>>>Medulloblastoma in terms of most common Posterior Fossa Tumor

What benign tumor is kids/younger adults have elongated Eosinophilic hairlike fibers (Rosenthal fibers)?

Pilocytic astrocytoma

Where does Polio hit?

Polio will damage the Anterior horn of LMNs S+S: asymmetric flaccid paralysis, hyporeflexia

Man in MVA sustains a closed-head injury. Has hx of alcohol, cocaine abuse, and HTN. Pt is comatose on exam with fixed pupils and "rigid extension of UEX and LEX." Damage to what brain structure causes this abnormal posturing?

Pons Key: Damage to Brainstem below Pons causes "Extensor Posturing" (Marykutty George!) In contrast, damage above Pons causes Flexor posturing.

Poorly soluble gas anesthetic

Poor solubility means 1. Low blood/gas partition coefficient 2. It saturates in the blood quickly leading to a fast rise in Partial Pressure (bc it's not soluble) 3. Increased P.P. allows drug to quickly speed to the brain, saturating the brain. So low solubility means quicky onset time Ex: NO (poorly soluble, has a blood/gas partition coeff of 0.47) On graph, the P.P. of NO would rise rapidly in blood and platueu, meaning blood quickly becomes saturated with the NO, allowing it to then quickly travel to brain and work fast. So: poorly soluble gas needs small amount to saturate blood rapid rise in P.P. in blood Rapid equilibration with brain rapid onset of action

What artery in Circle of Willis is associated with CN 3?

Posterior Communicating Artery is associated with CN3 Palsy. (via Saccular Berry Aneurysms)

2 things that cause Posterior column dysfunction of spinal cord?

Posterior column dysfunction of spinal cord occurs with either: 1. tertiary syphillis (tabes dorsales) 2. Vit B12 deficiency S+S: ataxia, decreased proprioception, decreased vibration, hyporeflexia.

27 y/o woman with severe, unilateral throbbing headache few times a month. Headaches cause phono/photophobia, N/V. Dr. gives a med and tell her to use it immediately during an attack. MOA?

Postsynaptic Serotonin Receptor Stimulation Key: Woman is having a classic migraine headache, (unilateral throbbing headache causing phono/photophobia and N/V) Tx of Migraines: Triptans (which a Serotonin 5-HT agonists), which stimulates postsynaptic Serotonin receptor. So, during a migraine, take a Triptan to "abort" the migraine.

What is potency of a gas anesthetic det by?

Potency is depended on MAC (Minimal Alveolar Concentration) MAC= conc. of gas needed in lungs to produce the desired effect in 50% of pts. Key: A "Potent Anesthetic" requires a very low MAC Overall, a Potent anestheti would have 1. low solubility, 2. fast rise in blood sat, and P.P. 3. Speedy brain equilibration and onset of action 4. low MAC needed by lungs to produced desired effect in 50% pts.

Organophosphate poisoning?

Potent AchE inhibitors (Insecticide organophosphates) prevents breakdown of Ach in NMJ. Causes too much PNS activity, and muscle weakness, fasciculations, paralysis, lethargy, seizures. Too much miosis, bradycardia, salivation, lacrimation.

What is the only med that helps reverse BOTH the Muscarinic and Nicotinic effects of insecticide posionsing?

Pralidoxime is the only med that reverses both Muscarinic and Nicotinic PNS effects of Organophosphates. MOA: Pralidoxime will restore AchE, which will then allow decrease in Ach levels. Note: Atropine can only inhibit the Muscarinic effects (M3). So it wouldn't help decrease the muscle paralysis/cramps of insecticide/pesticides poisoning.

Man with advanced HIV, hospitalized with new-onset headaches and peronality changes. Solitary mass in brain in Temporal lobe, due to EBV. Pt suffers from what?

Primary CNS lymphoma Key: EBV commonly attacks immunocomp (HIV/AIDS) pts and causes Primary CNS lymphomas. These tumors arise from B cells (CD20), and are due to EBV. High grade tumors, very very poor prognosis.

What is Gliosis?

Proliferation of Astrocytes in an area of Neuron degeneration is called Gliosis. Reactive Gliosis occurs to form a "glial scar", in order to compensate for the volume loss that occurs after neuronal death.

Med for essential tremor?

Propanolol

Short acting IV anesthetic that potentiates action of GABA at GABA-A receptor?

Propofol is a short-acting IV anesthetic that potentiates action of GABA at GABA-A receptor. can causes significant systemic vasodilation and hypotension though.

52 y/o woman has a hand tremor. Most prominent with activity like drinking from a glass or pouring some tea from a kettle. Pt says tremor gets better with small glass of alcohol. Best tx?

Propranolol Key: Lady has Essential (Familial) Tremor, is aka Postural Tremor. Essentially occurs with activity. A.D. inheritance. #1DOC: nonspecific Beta-blockers (Propranolol) Note: S+S get better with alcohol!! Intention=Essential=Postural Tremor

Infant born to 23 y/o mom is diagnosed with an inherited condition that impairs transport of Ornithine from cytosol to mitochondria. What can he not eat in diet?

Protein Ammonia is made from amino acids and is converted into urea through Liver (Urea Cycle.) Key: Ornithine transports into mitochondria in order to form urea. Ornithine combines with Carbomyl phosphate within mitochondria to form Citrulline in Urea Cycle. So: defect in urea cycle causes neurological damage due to accumulation of Ammonia, causing CNS dysfunction. Must restrict protein.

Supracondylar humerus fracture?

Proximal Median nerve injury S+S sensory loss in first 3 digits, also weak flexion and weak wrist ABduction Key: Wrist ABduction occurs via Median nerve

Pt with Chest pain comes to ER. Has history of HTN, asthma, and DM. Smokes 2 packs a day In ER, he is given a med that decreases his chest pain, but causes SOV. Physical exam reveals prolonged expirations and wheezes in bilateral lung fields. The med he was given for chest pain is impairing what?

Pt has DM, HTN, smoker, and angina, so he probs has: Atherosclerotic Coronary Artery Disease. The med he was given fixed his chest pain, but worsened his asthma. Beta blockers are given to patients with acute coronary attacks. Beta-1 Blockers will decrease HR, and Beta-2 Blockers will cause bronchoconstriction and wheezing. So: In this patient, blocking Beta2 will increase asthma/COPD S+S.

62 y/o man brought to ER with generalized tonic-clonic seizures. Has headaches, worse at night. Admitted to hospital, where he develops aspiration pneumonia and septic shock and dies. Brain section obtained post-mortem shows:

Pt has Glioblastoma Glioblastoma is the #1MCC of primary cerebral neoplasm of adults, located in cerebral hemipsheres and CROSSES midline ("Butterfly Glioma") Gliomas are highly malignant and contain necrosis and hemorrhage. Pts die less than a year after diagnosis.

Gram stain of CSF of 21 y/o male army recruit with fever, headache. Shows Bean-shaped Gram- diplococci in pairs. Route of entry?

Pt has Meningitis: N. meningitis enters via oral respiratory droplets route--->pharynx--->blood--->choroid plexus--->meninges

Man returns from camping trip 3 weeks ago. He now has severe agitation, disorientation, hallucinations, photophobia. Is also having pharyngospasms, choking and drooling. Several days later, he goes into coma and dies. What VIRULENCE factor is responsible?

Pt has Rabies (Rhabdovirus) virus infection (ssRNA virus), which binds to Nicotinic Ach receptors. Once in the body, Rabies stays local for a few weeks before binding to Ach receptors in PNS and travels retrograde to CNS. Note: Post-exposure prophylaxis is no longer helpful!!

What does above pt have?

Pt has a classic Thymoma, causing MG. So, he has Ab against neurotransmitter receptor (Ab against Nicotinic Ach receptors) MG causes EOM weakness, causing ptosis, diplopia. MG originates from Thymoma (anterior mediastinal mass on CT)

4 y/o boy has headache, especially when lying down. Has N/V, in mornings, blurry vision especially when looking up, and bilateral papilledema, inability to gaze upwards, and bilateral eyelid retraction. Brain mass is suspected. Location of lesion?

Pt has a pineal gland mass, causing obstructive hydrocephalus from aqeuductal stenosis. Causes papilledema, headache, and vomiting. Hallmark: Parunaud Syndrome (dorsal midbrain syndrome), where there's direct compression of pretectal region of midbrain. Limitation with gazing upward, bilateral lid retraction (eyelids way up). Key: Most common pineal gland mass is a "Germinoma" more common in boys and present with obstructive hydrocephalus and Parinaud (dorsal midbrain) syndrome.

46 y/o man comes to ER with double vision, ptosis, difficulty swallowing, nausea, dry mouth. 3 hours later, his wife experiences similar affects. Attended a picnic where they ate potato salad from home-canned potatoes. What step in neuromuscular transmission is affected?

Pt has classic Botulism toxin (via adult preformed toxin), causing ptosis, and 3 D's: Diplopia, Dysphagia, and Dysphonia, and then descending paralysis Botulism toxin inhibits Cholinergic nerves; both Nicotinic and Muscarinic motor neurons. Enters nerve terminals through endocytosis, and prevents BINDING and FUSION of Ach containing synaptic vesicles with plasma membrane. This blocks Ach release into synapse. Note: Botulism is Heat Labile (so CAN be killed by thoroughly heating food before eating it)

54 y/o man difficulty speaking. Responds slowly, becomes frustrated. Can grasp a pen with no problem, but has trouble signing name. Lesion where?

Pt has classic Broca's Aphasia (Motor Aphasia) Key: "Bocca" means mouth. Pt will have hard time speaking (dysarthria) Lesion: Inferior Frontal Gyrus of Left (Dominant) Hemisphere Pt can understand language, but have trouble forming words and writing (bc these require proper motor command)

45 y/o man with severe stabbing pain in extremities. Stumbles a lot. CSF shows +VDRL (Cardiolipin). Degeneration of what spinal cord area? TQ: Shows a cross-section

Pt has classic Tertiary Neurosyphillis (Tabes Dorsalis) Tabes Dorsalis causes degeneration of DCT in spinal cord. DCT damage causes: loss of vibration, proprioception, and ataxia. Also causes stabbing pain. Also have: Argyll Robertson pupils, and +Romberg sign

Highly agitated 54 y/o man unable to communicate. His speech is actually clear and "with conviction" but his sentences are totally incomprehensible. Doesn't understand the Doc's questions. can't follow oral or written instructions, and can't repeat phrases. What artery has a branched occlusion?

Pt has classic Wernicke's Aphasia due to occlusion of: MCA Key: Lesion in Wernicke's Aphasia causes: clear speech that is illogical (word vomit), and total lack of written and oral language comprehension. Also can't repeat stuff. (Wernicke=Word Salad)

Pain in right hand, causing diminished sensation in first 3 digits. Preserved sensation in thenar eminence. Flexion of wrist causes pain. Injury to nerve that courses through what?

Pt has damage to Median nerve (Pope's Blessing), which courses between humeral and ulnar heads of Pronater teres muscle, and then runs between: Flexor digitorum superficialis and Flexor digitorum profundus, before crossing wrist and carpel tunnel.

66 y/o man is Right-handed. Brought to E.R. with Left-handed weakness. Has left hand clumsiness, profound weakness affecting left arm. History of HTN and smokes cigarettes for past 40 years. P.E. shows Left facial weakness sparing the forehead, decreased L UEX muscle strength. What artery is occluded?

Pt has stroke, causing occlusion of Right MCA (Middle Cerebral Artery) Key: MCA occlusion causes CONTRALATERAL hemiplegia of face (spares forehead), and upper limb. Lower limb is spared. If the occluded MCA is in the Dominant (Left) hemisphere, can also cause Aphasia.

Depressed and suicidal woman rushed to hospital after sister finds her agitated and confused. Tremors, abdominal cramps, diarrhea. Pupils are dilated and she is diaphoretic. Bilateral hyperreflexia and ankle clonus. Increase in what in the CNS causes these S+S?

Pt is having classic: Serotonin Syndrome (confusion, agitation, tremor, tachycardia. HTN, clonus, hyperreflexia, hyperthermia, and sweating) Any drugs that increase Serotonin (SSRI + MAO inhibitors), or just really high doses of SSRIs cause this. Key: Tryptophan is the precursor to Serotonin.

62 y/o woman comes to ER with difficulty walking, started 3 hours ago. Pt cannot feel right side of body. Has history of HTN and DM. Smoked a pack of cigarettes for past 30 years. Loss of touch, temp, and vibration senses affecting Right upper and lower extremities. Also has loss of sensation over right side of face. She suffered from stroke where?

Pt suffered from stroke to Ventral posterior thalamus. VPT has the: Ventral Posterior Lateral nucleus (receives input from Spinothalamic tract and DCT) and Ventral Posterior Medial nucleus (receives input from Trigeminal pathways). Damage to VPT results in complete contralateral sensory loss.

Cause?

Pt with MI has liquification necrosis to lipid brain, associated with complete LYSOSOMAL digestion and removal of necrotic tissue. Forms a cystic cavity. Key: Hypoxic CNS injury is often followed by liquification necrosis. Also, an abscess can form in brain from fungal/bacterial infection and cause liquification necrosis of brain as well.

61 y/o man brought to ER after being found by paramedics. Smells of alcohol, and has vomitus in mouth. He goes into agonal (gasping) breathing and PEA. Dies. Autopsy shows death was due to vomitus occluding airway. Also, the brain shows foci of hemorrhage and necrosis in Mamillary bodies and 3rd and 4th Ventricles. Pt's brain has decrease in what?

Pt's brain would have a decrease in: Erythrocyte Transketolase activity Key: This alcoholic man had Vit B1 (Thiamine) deficiency, which is needed to make ATP. Thiamine is also needed by enzymes as a cofactor, to use Glucose for energy in the CNS. These enzymes include: 1. Pyruvate dehydrogenase 2. Alpha-Ketoglutarate dehydrogenase 3. Transketolase

28 y/o woman has spontaneous vaginal delivery under an epidural anesthesia. Since then, she complains of frequent fecal incontinence since delivery and mild perineal pain. Rectal exam shows decreased anal sphincter ton and loss of anal "wink" reflex. Cause of her fecal incontinence?

Pudendal nerve injury Key: injury to pudendal nerve occurs due to stress placed on pelvic floor during labor. Injury inhibits anal urethral and anal sphincters, leads to weakness in perineum, causing fecal/urinary incontinence. Note: Can also cause E.D. in men.

34 y/o man with weakness, dizziness, and paresthesia in face and extremities. Ate a Japanese pufferfish. Reduced muscle strength, decreased DTR. What part of action potential is impaired?

Pufferfish inhibits Na channel opening, so inhibits Na influx, and the depolarization phase of action potential

54 y/o white male dies of progressive Neurological disorder. Which part of brain would show cystic degeneration?

Putamen Key: progressive neurological disorders causing "cystic degeneration" of Putamen = Wilson's Disease!

Pyoderma gangrenosum

Pyoderma gangrenosum is assoociated with UC is a condition that causes tissue to become necrotic, causing deep ulcers that usually occur on the legs. When they occur, they can lead to chronic wounds. Ulcers usually initially look like small bug bites or papules, and they progress to larger ulcers.

C-spine injury causes?

Quadriplegia

Mutation of RB1 tumor suppressor gene?

RB1 gene mutation occurs on Chromosome 13 Causes Retinoblastoma, Osteosarcoma

Which organelle is the sight of synthesis of Secretory Proteins?

RER is site of synthesis of 1. Secretory proteins 2. Integral membrane proteins of Nucleus 3. Cell membrane integral membrane proteins 4. Lysosomal proteins.

How is Resting Membrane Potential in cell maintained?

RMP is equal to -70mV. Maintained by high membrane K+ permeability and low Na+ permeability.

76 y/o comes to office with Severe Insomnia for past few months. Best med?

Ramelteon Benzos, antihistamines, and sedating anti-depressants should be avoided in treatment of insomnia in old people due to adverse effects. Ramelteon= Melatonin supplement is best for insomnia in elderly.

29 y/o woman comes to office with GAD. Fluoxetine is given. MOA of Fluoxetine inhibits reuptake of n.s. released by specific set of neurons located where?

Raphe nucleus Pts with GAD should be given SSRI 1st line (Fluoxetine) In CNS, Serotonin releasing neurons are in Raphe Nuclei of Brainstem. Raphe nuclei are located in midbrain, pons, medulla. SSRIs will inhibit reuptake of Serotonin released from Raphe nuclei.

What happens with rapid correction of Hypernatremia?

Rapid correction of Hypernatremia can cause Cerebral edema bc water will try to dilate brain, causing it to swell too much.

Later, after neuronal cell death, the Astrocytic process forms what?

Reactive Astrocytosis forms a closely connected, firm meshwork called a Gliotic scar.

1-2 wks s/p I.S.

Reactive Gliosis via Astrocytes, vascular proliferation around necrotic tissue

Ischemic stroke Day 1

Red red neurons (Intensely Eosinophilic cytoplasm, nuclear pyknosis, loss of Nissl RER) aka Necrosis starts

42 y/o woman has double vision for few months, gotten worse in last 2 weeks. Hard time chewing, and avoids eating tough foods like steak. S+S improve with rest. What changes occurred to her post-synaptic muscle cell?

Reduced motor end plate potential Key: Woman has M.G. (Ab against Nicotinic Ach receptors). This inhibits ability of Ach to bind to and open postsynaptic cation channels, which in turn decreases the end-plate potential, preventing formation of a Muscular Action Potential. Note: End-plate potential must be reached (cross threshold) in order for Action potential propagation to occur.

26 y/o brought to ER with acute-onset diplopiam blurry vision. Comes and goes. Impaired ADDUction of Left eye with Right lateral gaze. The diplopia results from what?

Reduced saltatory conduction Key: Pt has MS (relapsing-remitting CNS disorder) causing recurrent demyleination, following by remyelination. M.S. has the INO and optic neuritis. Demyleination will impair speed of Saltatory conduction S+S of MS: heat intolerance, optic neuritis, INO, MLF probs, fatigue, bowel/bladder dysfunction. Diagnosis of MS = MRI Also, CSF shows IgG oligoclonal bands. Also contain plaques (lipid laden macrophages containing products of myelin breakdown), as well as demyleination with relative preservation of axons

MOA of Diphenhydramine (Benadryll)?

Reversible H1-receptor blocker. Sedating anti-Histamine used during night time to allow easier sleepiness and increase depth of sleep.

Function of ribosomes?

Ribosomes are present in cytoplasm are required for translation of mRNA into protein. rRNA in ribosomes engage mRNA and help tRNA enter during formation of polypeptide chains.

23 y/o comes to ER with tonic-clonic seizures. Dr gives her stat Phenytoin. However, her plasma levels of Pheytoin are low. What drug is responsible?

Rifampin Key: Metabolism of Phenytoin requires Hepatic 450 Oxidase and is dose dependent. Drugs like Phenobarbital, Carbamazapine, and Rifampin enhance the Phenytoin metabolisms and decrease its serum concentration bc of increases production of hepatic P450 oxidases.

Occlusion of ACA (Anterior Cerebral Artery)

Right ACA stroke causes: 1. Left sided severe weakness of face, tongue, 1. Left sided hemiplegia and maybe some spastic paralysis that affects L. LEX most. 2. Bilateral ACA stroke can cause behavioral problems (abulia= lack of willpower or motivation, primitive reflexes like moro/grasp can return), urinary incontinence

59 y/o man comes to office with diplopia and droopy Right eyelid. Dilated Right pupil, nonreactive. When asked to stare straight ahead, Right eye goes inferiorly and laterally compared to Left eye. What artery and what muscle is damaged?

Right Posterior Communicating Artery Damage to CN3 causes loss of everything except LR6 and SO4. Key: Again, this pt has Oculomotor CN 3 palsy (ptosis, down and out, dilated pupil) Why down and out? Bc of unopposed action of LR6 and SO4. (Posterior Communicating Artery)

Most common cause of Subarachnoid hemorrhage?

Rupture of Saccular (Berry) aneurysm, typically near Circle of Willis! KEY: Berry aneurysms will occur in Ehler's-Danlos Syndrome and AD Polycystic Kidney Disease.

43 y/o woman is brought to ER with sudden onset severe headache. BP is 160/90 and pulse is 90/min. Condition is caused by what?

Rupture of Saccular Aneurysm Key: woman had a SAH (excruciating head pain) which occurs due to Berry or due to AVM. Note: Berry aneurysms are seen in ADPKD, Ehlers-Danlos Syndrome. Lumbar puncture of CSF to see blood, but a noncontrast head CT is the actual Diagnostic test.

Vagal nerve damage

S+S: hoarseness, dysphagia, bad GI motility, tachycardia

LBP in lower back with flexing of back and raising legs. Pinprick to perianal area doe NOT cause rapid contraction of anal sphincter. What nerve roots is affected?

S4 Cauda equine syndrome: LBP radiating to legs, saddle anesthesia, impotence, loss of anal reflex, bowel and bladder dysfunction (S3-S5 nerve roots). Can also cause loss of Ankle-jerk reflex (S1 plantar flexion and inversion) This patient has cauda equine syndrome from S3-S5 and damage to pudendal nerve (innervated perineum)

56 y/o man with ADPKD presents to ER with sudden onset severe headache, with nuchal rigidity. Dx?

SAH Key: Saccular (Berry) Aneurysms in Circle of Willis are often seen in pts with ADPKD. Rupture causes classic SAH. Note: The nuchal rigidity can occur bc the blood could irritate the meninges Key Imaging (and safest) for SAH: Immediate Non-Contrast CT of Brain. If the CT does not show a SAH, but you're still suspicious, then do a lumbar puncture to check for blood in CSF. (Blood in CSF is most Sensitive test for SAH)

Subdural Hematoma?

SDH located between Dura and Arachnoid. Cause of SDH: tear if bridging cortical veins

What is Subacute Sclerosing Panencephalitis?

SSP causes progressive dementia, spasticity, and seizures. MOA: very abnormal Measles virus persists in CNS, for years even after recovery from measles infection

Fluoxetine?

SSRI (Prozac)

Ex of SSRIs

SSRIs include: Fluoxetine (Prozac), Sertraline (Zoloft), Citalopram (Celexa) Both used to treat depression (Jennifer Cano!), anxiety, and even PTSD. S.E: Worsening anxiety/agitation, insomnia

MOA of SSRIs?

SSRIs will only inhibit reuptake of Serotonin into presynaptic cleft. SSRIs are antidepressants (Fluoxetine, Sertraline, Paroxetine)

Where do Saccular (Berry) Aneurysms hit the most?

Saccular (Berry) Aneurysms occur most at Watershed borders: (Anterior and Posterior Communicating Arteries) in Circle of Willis Ant. Communicating artery aneurysm: compresses central optic chiasm--->Bitemporal Hemianopsia Post Communicating artery aneurysm: compresses Oculomotor CN3--->CN3 palsy= Dilated, Down and Out pupil.

Which brain tumor has "Palisading pattern" of nuclear-free zones.

Schwannoma

33 y/o pt with intracranial mass, composed of elongated cells with regular, oval nuclei. Classic "Biphasic pattern" of growth with dense cellularity interspersed with less dense more myxoid regions. The cells show "S-100" immunoreactivity. Dx?

Schwannoma Key: Hallmark of Schwannomas are the classic "Biphasic pattern" of cellularity (Antoni A has elongated, oval nuclei cells and Antoni B has myxoid areas) Key: Schwannomas also are +S-100 (means they are derived from neural crest) with a "Palisading pattern" MOA: Schwannomas are tumors of the Peripheral Nervous System (PNS = Schwaan cells duh!) that are most common in CN8 Cerebellopontine angle. These are called Acoustic Neuromas (S+S: tinnitus, vertigo, sensorineural hearing loss)

Sciatic nerve injury

Sciatic nerve innervated posterior thigh (hamstrings) Splits the Sciatic nerve into the common peroneal nerve and tibial nerve.

lady with the M.G. is started on meds that improve S+S. However, she complains of abdominal cramps, N/V, and diarrhea. What to tx her with now?

Scopolamine (M3-receptor Antagonist) Key: Tx of M.G. requires AchE inhibitors (like Pyridostigmine), but pt is now experiencing GI side effects (due to too much excess PNS cholinergic stimulation of gut) So, tx her with some Scopolamine (M3-receptor antagonist) that will help reduce Ach action of organs where Ach action is mediated by M3 receptors (gut), but not affect skeletal muscle (Nicotinic receptors)

MOA of Metoprolol?

Selective Beta-1-blocker. Metoprolol is used to treat acute coronary symptoms, angina, HF, HTN, and cardiac arrhythmias.

29 y/o white female with decreased vision and pain around eye. Intention tremor of left arm. Due to?

She has M.S. due to autoimmune disease. She is presenting with optic neuritis and intention tremor. Waxing and waning neuro deficits in 20-30 y/o white women. Common in colder Northern European environments. Hallmark of MS: increased IgG in CSF, detected as "oligoclonal band on gel electrophoresis."

28 y/o woman with Type 1 DM has malaise, abdominal pain, and N/V. P.E. shows dry mucosal membranes, involuntary rhythmic jerking movements of RUX for 1 minute. She remains conscious during episode. Labs show: +Ketones in urine, +Glucose in urine. What kind of seizure?

Simple partial Seizures are either 1. Focal/Partial (one area of brain) 2. Generalized (involves both cerebral hemispheres) Focal/Partial is further divided into 1. Simple (no LOC) 2. Complex (yes, LOC, and can have lip smacking) Key: Seizures occur due to abnormal synchronous firing of neurons in Cerebral Cortex This puts episodic RUX rhythmic jerking movements, with no LOC, suggest she has a Partial/Focal Simple Seizure (originating in CONTRALATERAL motor cortex)

Loss of superior gluteal nerve?

+Trendelenberg Weak ABduction of Ipsilateral Gluteus medius, Gluteus minimus, Tensor fascia lata Standing on Right leg, will cause left hip to drop. Lean towards ipsilateral (Right side)

if RMP of a nerve is -70mV, what would the RMP be if neuron was exposed to drug that activates GABA-A receptors, as an anti-seizure drug?

-75mV. Key: An anti-seizure drug that activates GABA-A receptors would increase either duration or frequency of Cl- channels, leading to increased passive transport of Cl- INTO cell. Causes hyperpolarization of cell (bc cell is now more negative than RMP).

23 y/o with myoclonic epilepsy of recent onset. P.E. shows proximal muscle weakness. Muscle biopsy shows muscle fibers with a blotchy red appearance. No family history bc pt is adopted. Probability that his offspring will inherit disease?

0% Key: "red ragged muscle fibers" are hallmark of mitochondrial diseases. Mitochondrial myopathy: Muscles look "red and ragged/blotchy" bc abnormal mitochondria accumulate under the sarcolemma. Pt's offspring CANNOT inherit disease from him bc this is strictly maternally inherited.

Acantholysis vs Acanthosis vs Dyskeratosis

1. Acantholysis= loss of cohesion between keratinocytes in epidermis. Acantholysis occurs in Pemphigus Vulgaris

Tx for insecticide posioning?

1. Atropine (M3-antagonist that will block the Ach receptors) 2. Pralidoxime (reactivates AchE by detaching the Organophosphates from the AchE to inhibit their inhibitory effect on AchE)

2 meds to give in Organophosphate poisoning?

1. Atropine (M3-receptor antagonist) 2. Pralidoxime (reverses BOTH Muscarinic and Nicotinic overstimulation, by actually "regenerating" AchE in NMJ)

2 abnormalities that can occur due to failure of processus vaginalis to obliterate?

1. Congenital hydrocele 2. Indirect inguinal hernia

PICA stroke (Lateral Medullary Syndrome)

1. Contralateral Body loss of pain and temp 2. Ipsilateral face loss of pain and temp 3. Ipsilateral Horner's Syndrome 4. Ipsilateral Cerebellar defects like ataxia. 5. Hoarseness, dysphagia, loss of gag reflex, vertigo, N/V.

Examples of lysosomal storage dx?

1. Gaucher's Dx: Accumulation of Beta-glucocerebroside 2. Neimann-Pick (Sphingomyelin accumulation) Note: These can also cause a cherry red spot in eye like Tay Sachs

What are the 3 mitochondrial diseases?

1. Leber hereditary optic neuropathy; causes bilateral vision loss 2. Myoclonic epilepsy (with ragged-red fibers)= causes myoclonic seizures, myopathy, associated with exercise. 3. MELAS (mitochondrial encephalopathy, lactic acidosis, strokes) Increased serum lactate levels.

What two things can lead to Cholinergic overstimulation?

1. Organophosphate poisoning (irreversibly inhibit AchE) 2. Pharmacologic Cholinomimetics (Carbachol, Methacholine)

High blood solubility

1. high blood/gas partition coefficient 2.Bc it dissolves in blood due to its high solubility, the blood saturation and rise in P.P. is delayed 3. Leads to slow travel to brain, increased onset time to work So: Highly soluble gas 1. needs larger amount in order to saturate blood and "level off" 2. Slow rise in P.P. in blood (so have to add more of drug so it will saturate blood) 3. Slow equilibration to brain 4. slow onset of action.

How to treat P.D.?

1st line includes: Selegline ( a MAO, type B inhibitor) which inhibits breakdown of DA. Selegine is used clinically to delay progression of P.D.

Kleinfelter's Syndrome?

47 XXY intellectual disability, sparse body hair, gynecomastia, cryptorchism (undescended testes), infertility.

When does caseous necrosis occur?

A focal T.B. infection in the CNS causes caseous necrosis. S+S: caseous necrosis looks "cheesy white/tan" and contains a ton of macrophages and other inflammatory cells, ultimately forming a granuloma.

Imprinting

A kid has expression of only one parental allele, while the other parental allele is deleted. The expressed parental allele undergoes mutaion. Causes PWS, AS. PWS: Paternally-inherited deletion of Chrom 15. Maternal allele is not even expressed. AS: Maternally inherited deletion of Chrom 15, paternal allele is not even expressed.

Conduction Aphasia?

A lesion affecting the (AF) Arcuate Fasciculus Speech is fluent (motor is fine in Inferior Frontal Gyrus) Can comprehend well (sensory is fine in Temporal Lobe) AF problems impairs: repetition!

S+S of Marfan's

A.D. Defect in Fibrillin tal, thin, aortic dissection, MVP, ectopia of lens, pectus excavatum, arachnodactyly, hypermobile joints.

23 y/o white male develops involuntary movements. What area of brain?

A.D. Huntington's Disease affects: Caudate

What are pts with NF1 at risk for?

A.D. condition, increased risk of CNS neoplasms (optic gliomas, pilocytic astrocytomas) causing increased ICP and headaches.

Marfan's Syndrome?

A.D. disorder of CR. Tall, arachnodactyly (long and thin fingers), lens dislocation, dilation of aorta.

What is the cause of Familial Hypercholesterolemia?

A.D. disorder. Defect in LDL-receptors, leads to LDL uptake and severe elevation in total cholesterol and LDL levels.

ACA infarct

ACA infarct will cause CONTRALATERAL LEX weakness

Effect of AD PKD?

AD polycystic kidney disease (Adult form) causes cysts in kidney, cysts in liver, cysts in brain causes RF, LF, HTN, and intracranial bleeds due to ruptured Berry aneurysm.

Defect in Arcuate Fasciculus?

AF is the connection between Broca's and Wernicke's areas. Disruption of AF, causes Conduction Aphasia S+S: normal comprehension (Wernicke's) and normal speech (Broca's), but impaired repetition.

Mutation of APC gene?

APC gene mutation causes Familial Adenomatous Polyposis Colorectal Cancer. FAPPC (5 letters) occurs on Chromosome 5

What is the Arteriovenous concentration gradient?

AV conc gradient is the same as solubility High AV conc gradient means more solubility, means more anesthetic is extracted from blood--->body tissues. This increases time it takes for blood to become saturated, causing a slower onset of action.

#1MCC of Intracranial hemorrhage in kids?

AVM Brain AVM is #1MCC of intracranial hemorrhage in kids

Ab that prevent intestinal attachment would be useful in inhibiting what?

Ab that prevent intestinal attachment would be helpful in inhibiting organisms that invade via enteric (oral) route in GI tract Ex: Salmonella, Shigella, E. coli, Entamoeba histolytica.

What is Hydrocephalus?

Abnormal accumulation of CSF in brain. Due to impaired drainage of CSF. S+S: Macrocephaly, poor feeding, muscle hypertonicity, hyperreflexia. Ex of Causes of Hydrocephalus: Aqueductal stenosis, Chiari malformation

Dyskeratosis?

Abnormal, premature keratinization of keratinocytes. Dyskeratosis occurs in SCC

Acanthosis?

Acanthosis is the increase in thickness of the stratum spinosum. Ex: Psoriasis, seborrheic dermatitis, acanthosis nigricans

What meds inhibit Ach degradation?

AchE inhibitors (Physiostigmine, Neostigmine, Pyridostigmine, and Edrophoium) Key: Neostigmine and Pyridostigmne tx M.G. by increased Ach availability at NMJ. Edrophonium and Ice pack test diagnose MG bc it improves S+S

Man who had chemo for H.L has N/V and requires IV fluids supplementation. Area causing N/V?

Acute N/V from chemo drugs are due to stimul. of: Chemoreceptors in Area Postrema which is located in the actual Medulla (Brainstem) near the 4th Ventricle

64 yr old had problem with back pain and while he was swimming he felt sudden weakness in his both legs and numbness from buttocks to toe..Rectal tone absent..and no sensation in perineal area. no history of trauma, no loss of consciousness, no dysarthria..Foley catheter was placed and 800 cc of urine is put out. Next best step of management??

Acute compression of the cauda equina Physicians who evaluate low back pain must be able to recognize the signs and symptoms of this relatively rare but critical spinal syndrome and must expedite emergent evaluation Patients with neurologic deficits of the lower extremities, perianal region, scrotum, penis, bowel or bladder (or both) need further evaluation. Patients with bowel or bladder incontinence should be considered to have neurologic spinal compromise until proven otherwise and need emergent imaging studies, preferably MRI. If the diagnosis of CES is confirmed, surgical intervention should be done as soon as possible to prevent progression of neurologic symptoms and to allow maximum neurologic recovery

Tx for HSV?

Acyclovir is an antiviral medication used to treat HSV meningitis.

Pharm experiment where Drug A is administered IV to a pregnant dog. HR, BP, pupil size, and uterine contractions are measures. With Drug A, uterine contractions go down, and also pupil size increases (mydriasis). MOA of drug A?

Alpha AND Beta adrenergic agonists Key: Stimulation of Beta-2= causes tocolysis (inhibits uterine contractions) Stimulation of Alpha-1(causes contraction of ocular pupillary dilator muscle; causing mydriasis)

What are the alpha-adrenergic antagonists used for?

Alpha-adrenergic antagonists (like Phentolamine, and Phenoxybenzamine) inhibit alpha-receptors to thereby inhibit SNS activity. Used to tx PCC

Side effect of Alpha1-blockers (like Prazosin)?

Alpha1-adrenergic blockers (Prazosin) can cause peripheral vasodilation, resulting in postural hypotension and dizziness.

Pt who is homozygous for Apolipoprotein E-4 allele is at risk for what?

Alzheimer's Dementia (late onset) Key: Early onset Alz Dx: associated with mutation in APP (chromosome 21), Prensilin 1, and Prensilin 2, Late onset Alz Dx: Apo E4

How does Alzheimer's Dementia present? Treatment?

Alzheimer's Dementia presents as progressive pre-senile Dementia with cortical atrophy on MRI, BUTT: NO OTHER radiology or lab abnormalities. Treatment: Donepezil (AchE inhibitor), Vit E (antioxidant), NMDA-glutamate receptor antagonists (Galantamine)

65 y/o woman brought to clinic due to worsening memory probs. History of HTN. Two years later she dues of massive M.I. Congo red staining of brain shows Hippocampus and Cerebral arterioles to have patchy red deposits that turn yellow-green under polarized light. Pt suffered from what disease?

Alzheimer's Disease Key: Pt has Amyloid (abnormally folded insoluble protein) with classic Apple-Green birefringence, stained on Congo red and viewed under polarized light. Alzheimer's Disease is associated with Beta-Amyloid deposits in brain parenchyme (neuritic plaques) and walls of cerebral vessels (Amyloid angiopathy)

Before Alanine can be converted to Glucose, it's amino group is transferred to what?

Amino group is transferred to alpha-Ketoglutarate Key: Alanine is the major amino acid that transfers Nitrogen to Liver for disposal. During catabolism of proteins, amino groups are transferred to alpha-ketoglutarate to form Glutamate. Glutamate is processed by Glutamate dehydrogenase in liver to liberate Ammonia, which enters Urea cycle and forms urea, excreted in urine. (#1 way Nitrogen is excreted)

Man with HL comes to ER with Cryptococcal meningitis (diagnosed via India ink staining CSF). Tx?

Amphotericin B Hallmark of Crytpococcal meningitis: Lumbar puncture with low glucose and increased protein. Low leukocyte count.

Cause of Cerebral Amyloid Angiopathy?

Amyloid Angiopathy occurs due to Beta-Amyloid deposition in walls of small-to-medium sized Cerebral arteries. This causes vessel wall thickening and rupture. Key: When you hear "spontaneous lobar hemorrhage" think Cerebral Amyloid Angiopathy Most frequent in Occipital and Parietal Lobes

67 y/o woman comes to clinic with decreased vision in her Right eye. Has HTN/DM. Vision testing shows: Right nasal hemianopia, due to right internal carotid artery aneurysm. What visual pathway is damaged?

Aneurysm to Internal Carotid artery can impinge on Optic Chiasm. Causes damage to Optic fibers (uncrossed) on ipsilateral side. So lady has damage to Right Optic Chiasm causing her Right nasal hemianopia.

Antagonsism of what receptor causes sedation and weight gain?

Antagonism of H1 receptor causes sedation and weight gain.

12 y/o boy with headaches and visual probs, have progressively worsened. Has +N/V. Imaging shows intracranial calcified mass in brain. Mass is surgically removed, and it shows thick, brownish-yellowsih fluid rich in Cholesterol. Mass derived from what cells?

Anterior Pituitary. Key: boy has abnormal remnants of Oral Rathke's pouch (derived from Surface Ectoderm, at oral roof of mouth). Remnants of Oral Rathke's pouch cause a: Craniopharyngioma in kids Hallmark: "Suprasellar tumor in kids" with Calcified cysts containing cholesterol crystals.

How do anti-seizure meds work?

Anti-seizure meds include: Benzos and Barbs MOA: Benzos and Barbs act on GABA-A-receptors to INCREASE Chloride conductance by either frequency or duration.

How could antibiotics cause bleeding?

Antibiotics (like oral Penicillin V) could reduce intestinal bacterial load, leading to decreased Vit K synthesis. Causes more bleeding.

M3-antagonists (Atropine, Scopalamine) poisoning?

Anticholingergic effects: cutaneous vasodilation, anhydrosis, hyperthermia, mydriasis, delirium.

primary virulence factor of Step pneumo, H. influenza, and Neisseria?

Antiphagocytic capsule

How do Antipyretics work to reduce fever?

Antipyretics for fever include: Acetomenophen and Ibuprofen. MOA: Reduces PGE2, which actually decreases the "Thermoregulatory Set Point of Hypothalamus", to decrease temp.

homonymous hemianopia can occur when?

Any unilateral lesion beyond the Optic Chiasm (Optic tract, Lateral Geniculate Body, Primary visual cortex) ---> can all cause CONTRALATERAL homonymous hemianopia

When does non-communicating hydroceph occur?

Arnold Chiari Dandy Walker Due to obstruction of flow of CSF, causing enlargement of ventricles

Chaiari II Malformation?

Arnold Chiari (Chiari II) is much more severe. Due to downward displacement of Cerebellum and Medulla through Foramen Magnum. Causes Non-communicating hydrocephalus due to "aqueductal stenosis" Compression of medulla, pons causes respiratory compromise. Lumbar myelomeningocele, which causes lower limb paralysis.

Hallmark of Arnold-Chiari Malformation?

Arnold-Chiari Malformation (aha Chiari II) has the hallmark downward displacement of the Cerebellar vermis and tonsils, protruding down below the Foramen Magnum. Chiari II occurs with Spinal myelomeningocele

Old man has chest pain. Developed Right sided hemiplegia, hemisensory loss, and aphasia 4 months ago. ECG in ER shows V-Fib. Pt dies. Autopsy of brain would show what in the brain lesion?

Astrocytes Key: Several months-yrs after an ischemic brain infarct, necrotic area in brain from the stroke will magically turn into a Cystic cavity surrounded by dense fibers. This occurs via Astrocytes (Glial Scar)

What neuronal cells are comprised of GFAP?

Astrocytes are derived from the: Neuroectoderm. Astrocytes contain GFAP (Glial Fibrillary Acidic Protein) Function: Repair, Lots of Structural Support, BBB,

What migrates to area of injury in CNS within 2 weeks of injury?

Astrocytes in CNS migrate to area of necrosis within 2 weeks of injury. As necrotic tissue is resorbed, a cystic space forms which is then surrounded by Astrocytes. Enlargement and proliferation of Astrocytes around area of Necrosis is called: GLIOSIS (Glial scar formation).

Where is the #1MC location of Saccular Berry Aneurysms?

At junction between Anterior Communication Artery and Anterior Cerebral Artery Berry Aneurysms cause SAH (worst headache of my life) Common in EDS, ADPKD, HTN, smoking. S+S: Can compress on Optic chiasm and Cause Bitemporal Hemianopsia.

What area of brain is affected in Wilson's Disease?

Atrophy of Lentiform nucleus (GP and Putamen) occurs in Wilson's Disease (Hepatolenticular degeneration)

Neuro tests to assess attention/concentration in old people?

Attention/concentration can be asses by asking them to: recite the months backwards, spell "world" backwards, count down from 100 in intervals of 3 or 7, etc.

66 y/o man with dizziness, dysarthria, and bilateral limb ataxia. 50 pack yr history. Chest X-ray shows mass in Right lung. Dies, but autopsy shows Extensive "Cerebellar Purkunje cell degeneration." Neuro cause?

Autoimmune Key: Pt developed a Parneoplastic syndrome (from Small cell lung-Ca) due to production of hormone-like substances from the actual Tumor cell. Immune response (Autoimmune) Ab against Parneoplastic tumor cells in Cerebellum caused degeneration.

25 y/o with Right arm weakness. Injured Right shoulder in MVA, and can't really elevate arm. Prominent atrophy of Deltoid. What nerve is injured?

Axillary Key: injury to axillary nerve occurs #1MC in shoulder trauma (anterior dislocation, or humeral fracture) S+S of Axillary nerve injury: sensory loss over lateral shoulder, problems with DELTOID (weak shoulder ABduction)

Anterior horn cells of spinal cord in experimental animal have Cell Body rounding, peripheral displacement of nuclei, and dispersion of Nissl Substance to periphery of cells. What is this reaction?

Axonal reaction Key: Axonal reaction occurs in Wallerian Degeneration, where axon becomes cut off from its Cell Body. occurs after axon has been severed. Hallmark: Rounded cells, peripherally displaced nuclei, dispersed Nissl RER to periphery. More protein synthesis to try and regenerate severed axon.

35 y/o woman with spasms and difficulty walking. Diagnosed with MS. Developed incontinence, weakness and numbness in LEX. Has spastic paraperesis in abdomen. What med to give?

Baclofen Key: Baclofen is a GABA-B receptor agonist. Good for MS associated spasticity. S+S of MS: paraperesis, urge incontinence, spasticity (muscle stiffness, painful muscle spasms, scissoring gait, bladder/bowel probs)

What microbe causes pseudomembranous pharyngitis?

Bacterial exotoxin via Corynebacterium diphtheriae. Causes a gray, non-scrapable pseudomemb pharyngitis in throat. Synthesizes a potent Exotoxin that causes Cardiac and Neural toxicity.

Why do S+S of PV occur after hot shower??

Bc after a hot shower, PV can cause pruritis with intense itching due to release of Histamine from Basophils from heat.

How does N. meningitis and all encapsulated bacterium survive in circulation?

Bc the polysaccharide capsule allows survival in circulation, by inhibiting opsonization, phagocytosis, and clearance of organism Key: Most vaccines against encapsulated bacteria actually TARGET the polysaccharide capsule.

Hallmark of Craniopharyngiomas?

Benign "suprasellar tumors" in kids and appear as cords/nests of "palisading squamous epithelium"

Flumazenil

Benzodiazepine receptor antagonist

MOA of Phenytoin, Carbamazepine, and Valproic Acid?

Benzodiazepines which decrease Sodium current in cortical neurons. Inhibit the neuronal high-frequency firing by blocking Sodium (Na+) channels and increasing the refractory period of the neuron.

MOA of Benzos?

Benzos are sedatives/hypnotics that calm pts down. (anxiety, panic attacks, insomnia, etc) MOA: Increase frequency of CL- channel opening, which allows Cl- INFLUX at the GABA-A receptors.

Where do Beta-Blockers work ?

Beta-Blockers will cause POST-synaptic NE receptor blockade

34 y/o woman with hearing loss, has ringing noise in left ear. Hearing is diminished on Left side. Left-sided facial numbness, asymmetric smile, decreased corneal reflex. Intracranial mass located where?

Between Cerebellum and Lateral pons Key: You SHOULD NOT have gotten this wrong!! This stem is describing a classic Acoustic Schwannoma at Cerebello-pontine angle!!!!!!!!! S+S: Ipsilateral sensoneural hearing loss/tinnitus, vertigo, loss of facial sensation, Note: Bilateral acoustic schwannomas occur in NF2 (Chromosome 22)

Key: Pt has loss of Biceps Reflex

Biceps reflex occurs at C5-C6 C5-C6 allows communication of Biceps with Brachioradialis

Result of Craniopharyngioma?

Bitemporal hemianopsia (occurs due to kid Craniopharyngioma or Pituitrary Adenoma) from compression of Optic Chiasm in Sella Turcica

Man has blurred vision in both eyes, past 2 days. Worse when he reads, but not when he drives. Started taking Diphenhydramine a few days ago for seasonal allergies. P.E. shows clear drainage in nasal mucosa. S+S are due to blockade of what?

Blockade of Ach Key: Yes, the pts use of Benadryl (Diphenhydramine) will block Histamine receptors. However, his blurry vision is due to the fact that First-generation Antihistamines (Chlorphenhydramine, Dimenhydranate, Diphenhydramine) ALSO have: Antimuscarininc effects. So bc these Antihistamines also act as Anticholinergics, they will cause mydriasis/cyclopegia and blurry vision. (especially to close objects)

Blockade of what receptor causes orthostatic hypotension and dizziness?

Blockade of alpha-1 adrenergic receptor causes orthostatic hypotension and dizziness.

23 y/o male in Motorcycle accident. Skull X-ray shows fracture of Temporal bone. Where did blood accumulate, causing pt to loose consciousness?

Blood accumulated between Bone and Dura mater. (Traumatic Epidural hematoma)

MOA of Benzos and Barbs?

Both Benzos (IV Lorazepam) and Barbs (Phenobarbital) bind to ligand-gated Cl- channels of GABA-receptors. These substances enhance the inhibitory effect of GABA by increasing Postsynaptic Cl- influx.

What pathway are Beta-1 and Beta-2 receptors associated with?

Both Beta-1 and Beta-2 adrenoreceptors are associated with cAMP signal transduction pathway. Results in increased Protein Kinase A activation. Note: PKA is a "cAMP-dependent protein kinase"

MOA of Phenytoin and Carbamazepine?

Both are anti-seizure drugs that disrupt generation and propagation of action potentials in axon by blocking: Voltage-gated Sodium channels.

MOA of Botulism toxin?

Botulism Anaerobic neurotoxin will: MOA: Prevent actual release of Ach from presynaptic nerve terminals at NMJ. Key: Toxin inhibits BOTH: Muscarinic and Nictonic receptors. S+S: fixed dilated pupils, dry mouth, descending flaccid skeletal muscle weakness, and the 3 D's: 1. Diplopia 2. Dysphagia 3. Respiratory Depression

What is the actual #1MCC of intracranial tumors in adults?

Brain metastases are the #1MCC of intracranial tumors in adults. Ex: Mets from lung, breast, skin (melanoma), renal cancers are likely to mets to the brain.

What drug may precipitate opioid withdrawal S+S?

Buprenorphine Key: Buprenorphine is a partial opioid receptor agonist that binds with high affinity. In patient's on long-term opioid therapy (such as oral Morphine) taking Buprenorphine can cause withdrawal S+S. Why? Bc Buprenorphine is a partial mu-opiod receptor agonist that binds with high affinity and prevents binding of other opioid meds. SO basically it antagonizes the opioid effects/pain relief of other opioids the patient may be taking, causing withdrawal effects

Which drug can be used to treat Depression AND Smoking Cessation?

Bupropion MOA: Bupropion (Dave!) is a DA and NE reuptake inhibitor. Good to treat Depression as well as Smoking cessation.

What would drop in PaCO2 do to brain?

By hyperventilating, you're causing a drop in PaCO2. This causes vasoconstriction and decreased blood flow, and decreased ICP in brain. (Ex: can do this by decreasing the ventilator settings by lowering PaCO2) Hypercapnea, on the other hand, will cause too much PaCO2 buildup, causing cerebral vasodilation and too much ICP. So: combination of hypoxia/hypercapnea cause increased cerebral vasodilation, blood flow, and high ICP.

46 y/o man with Left upper limb weakness. Chronic neck pain. Imaging shows spinal cord compression. Image shows reflex that's absent. Which segments?

C5-C6.

Triceps reflex

C7-C8

30 y/o with ADPKD. Runs to ER with sudden onset severe headache and confusion. CT shows blood in Subarachnoid space. On 5th day of admission, pt complains of weakness in Right arm and leg. This all could have been prevented by tx him with what?

CCB This pt's blood on CT shows a SAH. Key: CCB (Epecially Nimodipine) are used to prevent cerebral vascular spasms following a SAH.. Otherwise, without the Nimodipine, pts with SAH develop vasospasms, and more neuro S+S like weakness in extremities. Note: The stem hinted that pt has SAH due to ruptured Berry aneurysm bc of his history with ADPKD. Pts with ADPKD can commonly have Berry aneurysms

Cellular receptor for EBV?

CD21

HIV receptor?

CD4 T cells and CXCR4/CCR5

42 y/o man has impaired vision, and rapidly develops progressive dementia, and myoclonic jerks. Lapses into coma, and dies in 6 months. Brain tissue sample shows uniform vacuoles between neuron cell bodies. Dx?

CJD (Prion's Dx) Causes rapidly progressive dementia, myoclonic jerks, and uniform cystic vacuoles in brain (aka Spongiform Encephalopathy) Cause: Abnormal protein (Prions) Key: CJD (aka Spongiform Encephalopathy)

Smudge cell

CLL

What CN exits skull via Jugular Foramen, courses over Levator Scapulae, and innervated SCM and Traps?

CN 11 (Accessory) exits via Jugular Foramen (9, 10, 11) courses over Levator scapulae and innervates SCM and Trapezius.

Motor innervation of tongue?

CN 12 (Hypoglossal nerve) allows for motor movement of tongue

CN3 palsy:

CN 3 innervates SR, MR, IR, and Inferior oblique. CN3 palsy causes vertical and horizontal diplopia, ptosis, and enlarged/nonreactive pupils. "Down and out pupil"

CN6 palsy

CN 6 (Abducens) innervates the LR. Palsy of CN6 inhibits you from looking laterally (ABduction). So it causes Horizontal diplopia and inward deviation of eye.

26 y/o man was in a motorcycle accident has has to get surgery for a Maxillofacial injury. Post-op, he has difficulty chewing, and jaw deviates to Right side when he's told to open his mouth. What nerve injured, and in what Foramina?

CN V3 (Mandibular division of Trigeminal nerve) is damaged. Exits skull through: Foramina Ovale CNV3 innervates (Branchial Arch 1) muscles of mastication (Masseter, Medial and Lateral Pterygoid, and TeMporalis muscles)

Which CN is covered by only Oligodendrocytes instead of Schwaan cells?

CN2 (optic) is exclusively made of CNS Oligodendrocytes

CN2 (optic) damage?

CN2 (optic) transmits visual info to brain. Damage causes loss of vision. Ex: Right optic nerve damage causes Right eye blindness

Taste in anterior 2/3 of tongue?

CN7 (facial nerve)

Function of CN7?

CN7 is a "Mixed nerve" with both Motor and Sensory effects 1.motor movement of facial muscles 2. PNS innervation of lacrimal, submandibular, sublingual salivary glands 3. taste in Anterior 2/3 tongue

Left Glossopharyngeal nerve (CN9) damage would cause?

CN9 (Glossopharyngeal) damage would cause: 1. loss of Gag reflex (afferent) 2. loss of sensation in upper pharynx (gagging), loss of taste and sensation in posterior 1/3rd of tongue, Tonsils, 3. loss of sensation in tonsils Note: CN9 exits cranium via Jugular foramen (CN 9-11)

3rd pharyngeal arch

CN9; stylopharyngeal impaired taste, impaired gag reflex

Most common form of Necrosis?

COagulative necrosis. This form of necrosis, however, actually preserves the architecture of the necrotic tissue.

38 y/o woman with migraines, severe headache, N/V, photophobia, neck pain. Best next step?

CSF analysis pt has classic bacterial meningitis. Evaluation requires prompt blood culture, empiric antibiotics, and lumbar puncture to analyze CSF. Bacterial meningitis causes increased inflammatory cytokines (IL-1, IL-6), leading to severe fever, inflammation, and injury to BBB. Causes increased brain edema, and increased ICP (causing headache, stiff neck, photophobia, and altered mentation.

Flow of CSF

CSF is made by 4 Choroid plexuses found in each Ventricle. Choroid plexus is made of Ependymal cells (made of tight junctions) 1. Once formed, CSF flows from Lateral Ventricle--->3rd Ventricle (via Foramina of Monroe) 2. 3rd Ventricle--->4th Ventricle via Cerebral Aqueduct of Sylvius 3. Exits 4th Ventricle---->Subarachnoid space via Foramina of Luschka (lateral) and Magendie (midline) 4. CSF then returns to CNS venous circulations via Arachnoid villi (granulations) So, 1. Obstruction of Foramen of Monroe would cause: obstruction only of Lateral Ventricle 2.Obstruction of Cerebral aqueduct would cause enlargement of 3rd Ventricle AND Lateral Ventricle 3. Obstruction of Foramina of L+M would cause enlargement of all 4 ventricles

Path of CSF?

CSF travels from: 1. Lateral ventricles--->Interventricular foramen of Monroe---> 3rd ventricle--->Cerebral aqueduct--->4th ventricle--->Foramina of Luscka and Magendi--->finally to the Subarachnoid space. In Subarachnoid space, the CSF is absorbed via arachnoid granulations--->enters venous sinuses. Choroid plexus is site of CSF secretion.

Ehler's Danlos Syndrome?

CT disorder due to defect in Type V collagen. Skin hyperelasticity, joint hyperlaxity.

Man has recent onset severe, throbbing Right-sided orbitofrontal headache, and double vision. Has poorly controlled HTN too, and is a smoker. Right pupil is dilated, and nonreactive to light. Right eye is "down and out" with ptosis. CT would reveal a large aneurysm where?

CT would reveal a large aneurysm to Right sided CN 3, associated with Right sided Posterior Communicating Artery aneurysm. Key: CN 3 palsy causes blown pupil (mydriasis) that's "down and out."

Global Aphasia?

Can be secondary to a proximal MCA occlusion affecting both superior and inferior division of the MCA Impaired everything! Can't talk, understand, or repeat shit!

Pt has sudden right sided severe facial pain. Pain is like a "knife stabbing her face." Pain lasts several seconds, usually after meal or brushing teeth. Best tx?

Carbamazepine Key: Trigeminal neuralgia (tic douloureux) presents as sudden severe "electric shock-like" pain or a "stabbing" pain. Occurs in CN V2 and V3. Carbamazepine is the #1 DOC for Trigeminal neuralgia. MOA: Carbamazepine, like Phenytoin, inhibits high-frequency firing by reducing ability of Sodium channels to recover after inactivation.

Most feared complication of Carbamazepine?

Carbamazepine is the #1 DOC for: Complex partial seizures Most feared side effect: Agranulocytosis or aplastic anemia.

Risk of Cardiac arrest after an acute MI?

Cardiac arrest (MI) can cause global ischemic hypoperfusion (ischemic hypoxic encephalopathy) affecting Hippocampus first. Can also affect Purkunje fibers of Cerebellum

15 y/o girl has clumsiness and gait instability. Has to hold on to a railing when she walks to avoid falling. P.E. shows kyphoscoliosis, pes cavus, and LEX ataxia. Impaired position and vibration sense. Greatest risk of dying from?

Cardiomyopathy Key: Girl has classic Friedrich's ataxia (A.R.) disease causing a progressive gait ataxia due to degeneration of Spinocerebellar Tract, impaired joint and vibration sense (due to degeneration of Posterior columns and DRG), and kyphoscoliosis, and pes cavus. Death is due to hypertrophic cardiomyopathy.

Cauda equina syndrome?

Cauda equina syndrome (CES) is a serious neurologic condition in which damage to the cauda equina causes loss of function of the lumbar plexus (nerve roots) of the spinal canal below the termination (conus medullaris) of the spinal cord. (Saddle anesthesia) CES is a lower motor neuron lesion. L5-S1 level

MOA of C. botulism toxin?

Causes ptosis and descending Flaccid paralysis by: Inhibiting release of Ach from the actual presynaptic neurons. (Flaccidity)

30 y/o man has hx of epilepsy. Neuroimaging shows focal lesion in Right motor area. Surgery is done to remove the lesion, and the excised 1.5 cm mass is composed of engorged purplish clusters of dilated capillaries. Dx?

Cavernous Hemangioma Key: Cavernous Hemangiomas are made of abnormally dilated caps, separated by thin C.T. layer. Have tendency to bleed, and also growth in brain can cause seizures.

Man with high grade fever, headache, double vision. Purulent nasal drainage and frontal headache. P.E. shows ptosis, mydriasis, proptosis, loss of corneal reflex in Right eye. Pt unable to move Right eye in any direction. Decreased sensation in R. upper face. What structure is involved?

Cavernous sinus Key: Infection of medial face, sinuses (Ethmoid sinus, Sphenoidal sinus) can spread through Fascial venous system into the Cavernous sinus. (Cavernous sinus thrombosis) S+S: fever, headache, proptosis, and IPSILATERAL defects of CN 3, 4, 5, 6). The Proptosis is a sign of eye protrusion due to impaired venous drainage. usually occurs from S. aureus infection in sinuses

Tx for H. influ associated pneumonia or meningitis?

Ceftriaxone

54 y/o comes to ER ith fever, chills, malaise for last 24 hours. Recently diagnosed with HL. Temp is 103. Blood culture shows motile, Gram+ rods, that produce narrow zone of Beta-hemolysis on blod agar. Bacteria grow well in fridge. MOA of elimination of bacteria from body?

Cell-mediated immunity Key: Pt has Listeria (Beta hemolytic on blood agar, tumbling motility at 22 degrees C, cultured at temps as low as 4 degrees C. T-Cell-mediated immunity is essential for elimination of Listeria from body. Neonates (up to 3 months) are very vulnerable to infection due to immature T-cell mediated immunity. Listeria stimulates cytokine production via T-cell CMI, leads to macrophages activation and killing of intracellular engulfed Listeria.

receptor for CMV

Cellular integrins

65 y/o male complains of bilateral visual probs. Fundoscopy shows small, yellow retinal lesions clustered in macula. What would you expect on visual exam?

Central scotomas Key: A scotoma is a visual field defect in retina or optic nerve. Causes focal areas of bad vision, surrounded by normal vision. Lesions of macula cause central scotomas. Note: Macula is located near the center of the retina. This older man probs has macular degeneration, common in oldies, with progressive loss of central vision due to deposits of fat behind retina.

4 y/o boy has difficulty walking. Past medical history involves frequent respiratory infections. Cultured cells shows high rate of radiation-induced genetic mutation. This patient is most likely to experience what?

Cerebella atrophy Pt has ataxia telengectasia (an autosomal-recessive disorder) results in defect in DNA-repair genes. (DNA of these patients is hypersensitive to ionizing radiation like sunlight, UV, gamma and X-rays, oxygen free radicals) S+S: cerebella ataxia, teleangectasias, respiratory tract infections, malignancy. Note: Telangiectasia= abnormal dilation of capillary vessels.

Tonsilar herniation?

Cerebellar tonsils displace through foramen magnum, and compress Medulla

75 y/o man brought to ER with vision and R. sided hemisensory loss. Head CT shows multiple, small, lobar hemorrhages of various ages in Occipital and Parietal lobes. Also has bleed in Left ParietoOccipital lobe. 2 years ago, same pt developed sudden Right sided arm weakness and had a small Left Frontal lobe bleed. Cause of all this shit?

Cerebral Amyloid Angiopathy Key: Cerebral Amyloid Angiopathy is the #1MCC of: spontaneous lobar hemorrhages, especially in Occipital and Parietal Lobes.

9 y/o girl has severe headache, lethargy, and N/V. CT shows mass lesion, enlarged Lateral and 3rd Ventricle, and normal-sized 4th Ventricle. Site of Obstruction?

Cerebral aqueduct Key: CSF flows from 3rd Ventricle---4th Ventricle via the Cerebral Aqueduct of Sylvius. Note: This girl has Non-communicating Hydrocephalus due to obstruction of CSF flow

If cerebral blood flow does not meet the actual cerebral metabolic demand, what occurs?

Cerebral ischemia occurs when blood flow to brain doesn't meet metabolic demand of brain tissue. Ex: Thromboembolism results in cerebral ischemia--->causes focal neuro deficits Note: Global cerebral hypoperfusion results in altered mental status, LOC, and even coma! Most common in ACA or MCA (Watershed Borders)

What aneurysms occur in pts with longstanding chronic HTN?

Charcot-Bouchard aneurysms occur in pts with chronic HTN history.

What does +Romberg sign test for?

Checks for intact Proprioception Key: Romberg test tries to differentiate between Cerebellar ataxia vs. Sensory ataxia. By having pt close eyes and stand with arms at side, if they can't maintain balance, this means they have +Romberg with sensory ataxia. Sensory ataxia occurs due to defect in Posterior Column or Peripheral nerves, which are damaged in Tabes Dorsalis Syphillis/Vit B12 deficiency. Key: Cerebellar ataxia have a -Romberg sign, bc they can stand still with eyes closed. Evaluate Cerebellar ataxia with rapid alternating movements, finger-to-nose test, heel-to-shin test.

65 y/o man undergoing rehab after stroke has weakness and increased tone in Left UEX and LEX. You give him Diazepam, to decrease spasticity. What drug must patient avoid if he is on Diazepam?

Chlorpheniramine Pts taking Benzos should avoid 1st generation H1-receptor antagonists (like Diphenhydramine or Chlorpheniramine), bc it can cause a ton of sedation, especially if combined with Benzos (CNS depression)

NF1?

Chromosome 17 von Reckinghausen's Dx S+S: cafe-au-lait spots Lisch nodules in eye multiple neurofibromas

Elderly male with tenderness over Temporal area, blurred vision, sudden onset severe headache, and intermittent jaw claudication?

Classic Giant cell Temporal Arteritis (often occurs in pts with Polymyalgia Rheumatica) Labs: High ESR Tx: Stat corticosteroids (don't wait for results!) to prevent blindness

Stroke to ACA would impair what action?

Climbing stairs Each ACA supplies superior and medial regions of ipsilateral hemisphere. Occlusion of ACA would affect sensory AND motor function of CONTRALATERAL leg and foot. (Ex: can't climb stairs) Key: ACA supplies Frontal and Parietal lobes

MOA of Clozapine?

Clozapine treats the "negative S+S of Schizophrenia" but can cause agranulocytosis and severe seizures.

When does Coagulative necrosis occur?

Coagulative necrosis is the #1MC necrosis. develops after ischemic injury, in most tissues (But NOT the brain). Key: In coagulative necrosis, due to hypokix cel death, it's good bc the tissue architecture is preserved, even after irreversible cell death. Cells become "enucleated," with eosinophilic cytoplasm,

MOA of Cocaine?

Cocaine blocks reuptake of NE, 5-HT, and Dopamine in CNS. Causes cardiac angina, cardiac ischemia, HTN, and seizures. Cocaine withdrawal causes severe depression

67 y/o man with Right-leg weakness and numbness. Can't evert the right foot. Numbness over dorsum of foot and lateral shin. Lifts right foot very high and slaps it to ground with each step. What was injured.

Common peroneal nerve injury occurs due to damage to fibular head. "Foot drop" and "Steppage gait"

What to prescribe for business man who complains of insomnia due to anxiety over a failing investment.

Commonly prescribed hypnotic agent that is anxiolytic (anti-anxiety), muscle relaxant, and anticonvulsant that're safe for insomnia= Benzos (like Triazolam) (increase the frequency of Cl- channel opening) MOA: allow more Cl- to come into cell, causing hyperpolarization of cell (more negative inside than RMP)

What would happen with complete transection of spinal cord?

Complete transection would cause complete and total bilateral loss of all sensation as well as all voluntary motion BELOW level of lesion.

Compression of optic chiasm causes?

Compression of optic chiasm causes Bitemporal hemianopsia, causing loss of vision in Temporal eye fields of both eyes. Ex: pituitary adenoma

21 y/o woman has headaches, dizziness, and gait imbalance history. Gets headaches in occipital region. MRI shows a low-lying cerebellar tonsils in foramen magnum. Cause?

Congenital defect Key: Lady has classic Chiari 1 malformation (congenital) due to underdeveloped Posterior Fossa. This causes Cerebellum and Medulla to herniate through Foramen magnum. Chiari II is much more severe, causing lumbar myelomeningocele and hydrocephalus.

Complex partial seizure

Consciousness is impaired Symptoms are dependent on the anatomical location of the seizure Most common location is in the: temporal lobes Key: These seizures may have an aura (or warning, which technically is itself a focal aware seizure). These seizures include automatisms (such as lip smacking, picking at clothes, fumbling), becoming unaware of surroundings, and wandering.

Simple partial seizures

Consciousness is spared Symptoms are dependent on the anatomical location of the seizure e.g., abnormal shapes or flashes when there is a seizure in the primary visual cortex Typically no postictal deficits in brief simple partial seizures

34 y/o man comes to Hx with hard time swallowing, dry mouth, and blurred vision. Has hx of depression. P.E. shows: Mydriasis, poorly reactive pupils. Decreased muscle action potential. Rapid nerve stimulation, however, helps increase muscle action potential. What's his problem?

Consumption of home canned foods. Key: Don't get tripped up by distracters!! (Like hx of depression, etc!) Key: Clostridium Botulism: Botulism toxin (caused by preformed toxin in adults) inhibits release of Ach from presynaptic nerve terminals. Toxin inhibits BOTH Nicotinic and Muscarinic receptors. S+S: ANS effects like fixed pupillary dilation, dry AF mouth, flaccid muscle weakness, diplopia and/or ptosis. Eventual descending paralysis leads to diaphragmatic respiratory paralysis.

Man with Generalized tonic-clonic seizures. Complains of intermittent problems with vision. Imaging shows mass in Right Temporal Lobe. What visual defect?

Contralateral Superior Quadrantanopia Key: Lesion to Temporal lobe will disrupt Meyer's Loop, and produce a Contralateral Superior Quadrantanopia Note: Lesion to Temporal lobe can also Cause Wernicke's Aphasia

Medial Medullary Syndrome?

Contralateral spastic paralysis, contralateral loss of vibration and proprioception, Ipsilateral flaccid paralysis of tongue (CN 12)

23 y/o girl's ex-fiance broke up the engagement 3 days ago, and now she has severe left leg weakness. DTR are +2. She doesn't like coming to the hospital. Diagnosis?

Conversion disorder which is basically neuro S+S that don't match with a neuro disease. Often acute onset, associated with acute stressors. Can involve paralysis, or weakness, or even seizures, etc.

What could occur with rapid correction of Hyponatremia?

Correcting electrolytes (low Na+) way too quickly can cause Osmotic demyelination syndrome in Pons aka Central Pontine Myelinolysis (Locked In Syndrome) What is it? Rapid correction of hyponatremia can cause osmotic demyelination of axons in central part of Pons. Locked in Syndrome= quadriplegia, dysphagia, dysarthria, wekaness.

Man with HIV has progressive memory loss, cognitive decline. Falls more frequently, is more tired. What would his CT show?

Cortical atrophy with Enlargement of ventricles aka Hydrocephalus ex-vacuo occurs due to enlargement of Ventricles in brain. Occurs not due to actual increase in CSF, but rather due to cortical atrophy which makes it seem like there's more CSF in brain. So net result: Normal CSF, cortical atrophy

Antidote for Serotonin syndrome?

Cyprohepatadine (an Anti-Histamine with anti-Serotonergic properties) used as an antidote for Serotonin Syndrome.

What happens if Phenytoin is admin with a CytP450 inhibitor?

CytP450 inhibitors: Isoniazid, Fluconazole, Cimetidine, PPI (Omeprazole), Macrolides, Azole (antifungus), and Grapefruit juice will decrease metabolism of Phenytoin bc they inhibit CytP450. Causes severe Phenytoin buildup and toxicity.

16 y/o boy with difficulty walking and tremors. Sleeping more than usual, poor appetite, tremor in both hands. Broad-based, unsteady gait. Labs show elevated Serum Transaminases and low Serum Ceruloplasmin. Tx?

D-penicillamine Wilson's disease causes cystic degeneration of Putamen. A.R. condition, with excess serum copper, copper in liver, brain (basal ganglia) eye cornea, urine,. Note: Zinc can also help bc it interferes with copper absorption. Elevated transaminases indicate liver injury.

8 y/o boy with hx of recurrent pulmonary infections. Unsteady gait, loses balance and falls a lot. Language development is also slow. Child's eyes and head "do not move smoothly" when he's looking at objects. Numerous superficial blanching nests of distended capillaries on sun-exposed skin areas. Genetic defect?

DNA break repair is defective Child has classic Ataxia Telangiectasia (Cerebellar ataxia, telangiectasia, increased URT infections) AR inheritance Defect: in gene that codes for "ATM" gene which plays role in DNA break repair. IgA deficiency, predisposes to infections of Upper and Lower Respiratory Tract. Key: DNA is pts with Ataxia Telangiectasia is hypersensitive to X-ray radiation that causes multiple chromosomal breaks.

55 y/o right handed woman complains of acute onset headache and bad vision. Becomes unconscious on the way to hospital. CT shows acute hemorrhage of Left temporal lobe, with compression against the Tentorium cerebelli. What CN is affected?

Damage to Left Temporal Lobe would affect the CN3 (Occulomotor nerve). Key: The dural folds of brain (Tentorium cerebelli and Falx cerebri) divide the skull into compartments. There is absolutely no room for brain expansion in event of tumor/hemorrhage. If parts of swelling brain protrude into Tentorium cerebelli, Falx cerebri, or Foramen magum: Herniation

65 y/o man has lung adenocarcinoma and abdominal mets. Develops small temporal lobe hemorrhage, and seizures. Brain starts to expand and herniate. Findings?

Damage to Temporal lobe will damage CN 3 Man is having an Uncal herniation due to increased ICP from hemorrhaging of intracranial mets in brain due to his lung cancer. Expansion of Temporal lobe will cause an Uncal herniation with compression of CN3. Causes ipsilateral CN3 (Oculomotor) nerve palsy with fixed "down and out" blown pupil.

2 y/o immigrant boy brought to office bc of irritability and excessive crying. Was delivered via C-section bc of obstructed labor. He is developing more slowly than his brother, and can't lift his head, crawl, or walk. His head is huge! CT shown: Dx?

Dandy-Walker malformation Key: CT shows Dandy-Walker Malformation. Occurs due to absence of Cerebellar vermis, and cystic dilation of 4th ventricle in Posterior Fossa. S+S: Infants have developmental delay, huge skull, ataxia (due to problem with Cerebellar vermis), non-communicating hydrocephalus.

Man brought to hx. Has back-to-back Generalized tonic-clonic seizures and never fully regains consciousness between episodes. IV Lorazepam stop the seizures, and another drug is also admin to prevent recurrence. MOA?

Decrease Sodium Na+ current in cortical neurons Key: You have to recognize this guy is having Status Epilepticus (seizures lasting >5 min/seizures back to back with LOC) Intial tx of Status epileptics: 1. IV Lorazepam to stop the seizures, bc it's a Benzo and potentiates inhibitory effects of GABA in CNS. Has rapid onset of action. 2. Phenytoin: long-acting anticonvulsant that inhibit neuronal high-frequency firing in Cerebral Cortex, by MOA: inhibiting ability of Na+ channels to recover from inactivation. Key: Must given both IV Lorazepam and Phenytoin at same time!!

72 y/o female suffers progressive memory loss over past 2 yrs. and now needs help with ADLs like shopping, housekeeping. MRI shows diffuse cortical atrophy. What biomechanical change contributes to S+S?

Decreased choline acetyltransferase activity in nucleus basalis Key: Pt has Alzheimer's Disease (with low Ach) Low levels of Ach in Hippocampus (which forms new memories) and something called the Nucleus Basalis (participates in memory and congition) Tx: AchE inhibitors

51 y/o woman whines about Right side of face feeling "funny." Has Type2DM. Asymmetry of face when she smiles, puffs out her cheeks, and closes eyes. Right mouth is drawn towards Left side. What other symptom would she have?

Decreased tearing from Right eye Key: Woman has classic Bell's Palsy (CN7 Palsy) S+S: decreased tearing, hyperacusis, loss of taste in Anterior 2/3 of tongue, Sudden onset unilateral facial paralysis, can't close eye, can't elevate eyebrow/forehead, can't smile or frown on affected side. Mouth is "drawn to NON-affected side"

8 y/o diagnosed with absence seizures, has been undergoing treatment with Ethosuximide. MOA of Ethosuximide?

Decreases Calcium current in Thalamic neurons key: Ethosuximide blocks the T-type Calcium channels that trigger and sustain rhythmical burst discharges in Thalamus.

64 y/o comes to office with 2 months of severe episodic facial pain. Pain is sudden-onset, severe, and electric shock-like pain. Pain is on Right side of cheek and ear, lasting several seconds. Things like shaving, or washing face can trigger pain. Pt is a veteran, with PTSD. Treated with a med for S+S, and comes back with improvement. However, CBC shows pancytopenia. MOA of drug given?

Decreases Sodium current on multiple levels This pt has Trigeminal Neuralgia (Tic Douloureux) with electric shock like pain in CN V. Key: 1st line tx of Trigeminal Neuralgia is: Carbamazepine. Carbamazepine inhibits neuronal high-frequency firing by inhibiting Na+ channels. However, it can cause severe Bone Marrow Suppression (low Hb, low RBC, low platelets, now Leukocytes, etc) So: If pt is given either Carbamazepine or Clozapine, MUST monitor CBC Clozely!

23 y/o woman with acute attack of optic neuritis has history of relapsing-remitting M.S. MRI of brain shows areas of demyelination involving Left optic nerve. Pupillary light reflex?

Demyelinated left optic nerve (CN2): 1. Shine light in right eye allows both Right and Left eye to constrict 2. Shine light in Left eye, no change in both Right and Left eye.

Where are "Demyleinated plaques" seen?

Demyelinated plaques are found in M.S.

What disease causes CD8+ T-cell mediated nerve fiber demyelinization?

Demyelinization of PNS caused by CD8+ T cells and macrophages, causing ascending paralysis is associated with classic GBS. (C. jejuni)

38 y/o woman comes to urgent care with 2 day history of sore throat. Has fever and cervical lymphadenopathy. Edema and erythema involving posterior pharynx, with purulent tonsilar exudate. +Streptococcal Ag test. She is immediately started on oral Penicillin V. But the next day she has skin lesions and a pruritic skin rash. Most likely finding on biopsy?

Dermal edema Key: Pt has Uticaria (which is nothing but dermal edema) due to a transient HSN reaction. Key: Uticaria occurs due to IgE-mediated (dependent) mast cell degranulation. Type 1 HSN reaction. Uticaria occurs due to increased permeability of microvasculature, leads to edema of superficial dermis.

MOA of urinary retention?

Detrusor muscle relaxation and contraction of external urethral sphincter.

Cause of Diabetic nephropathy?

Diabetic nephropathy occurs bc of "Glucose-induced neural toxicity" High glucose conc results in formation of glycosylation products (Ex: HbA1c) These NEG glycosylated products accumulate in kidney, nerves, and also Sorbitol accumulates within eyes and nerves, causing the nephropathy and retinopathy.

55 y/o man has right shoulder pain, can't ABduct it or Externally rotate it. In surgery he receives anesthesia with injection between Right Anterior and Middle Scalenes for blockade of brachial plexus. What muscle is paralyzed now?

Diaphgram Bc the anesthesia is going between the Scalenes to block the brachial plexus, you'd cause transient diaphragmatic paralysis as well bc the C3-C5 Phrenic nerve courses through the scalenes up there.

What Benzo is used to treat pt's who have both Seizures AND Alcohol Withdrawal (DT)?

Diazepam

Which Benzo is given for prolonged seizures (status epilepticus) lasting >5 minutes?

Diazepam S.E. abuse potential, dependence potential.

14 y/o boy has seizures that cause: blank stare, lip smacking, followed by Generalized shaking. Occasionally bites his tongue and feels confused afterwards. Dr. prescribes Lamotrigine. Boy should be warned to immediately contact Dr. if what happens?

Skin rash Key: Lamotrigine can be used to tx both Partial or Generalized seizures. MOA: Lamotrgine blocks the Voltage-gated Sodium channels. S.E.: Can cause SJS as well as Toxic Epidermal Necrolysis (TEN) Hallmarks of SJS or TEN? Flu-like S+S like fever, malaise, myalgias, followed by insane amounts of skin lesions on skin and oral mucosa.

21 y/o man has impaired balance, tremor, hard time speaking. Elevated Serum Transaminases. Siblings were dx at young age with progressive neuro dx. What diagnostic study to order to CONFIRM dx?

Slit lamp examination Key: Pt has early onset neuro decline due to Wilson's Dx (H.D. like tremor, and chorea, and ataxia) Wilson's is an AD disease, due to excess Cu2+ buildup in tissues (eyes, brain, liver) S+S: Liver failure, gait imbalance, hard time speaking, depression, personality changes, Kayer-Fishcer rings Key: The diagnosis can be confirmed via Slit-Lamp test which checks for the Kayer-Fischer rings Tx: D-Penicillamine (Copper chelator), or Zinc (imapirs with Cu2+ absorption)

LEMS MOA:

Small cell Lung-Carcinoma LEMS causes AutoAb against pre-synaptic Voltage gated Ca2+ channels. S+S: muscle weakness that usually starts in legs and S+S improve with post-excitation facilitation: improvement with rapid nerve stimulation.

Excess anxiety, pre-occupation with a bunch of somatic symptoms.

Somatic symptom disorder

What is the speed of onset of action determined by?

Speed of onset of action depends on rate at which brain tissue takes up the anesthetic (which basically depends on the solubility of anesthetic in the blood, speed of blood saturation, speed of rise in P.P.)--->speed of travel to brain

Mutation in Niemann-Pick Disease?

Sphingolipid degradation is defective in Niemann-Pick Disease. AR disease with defect in Sphingomyelinase, results in accumulation of Sphingomyelin within monocytes, leading to early childhood death. S+S: Hepatosplenomegaly, Anemia, Motor neuropathy causing Hypotonia and Areflexia, and "Cherry Red Spot" on Maculas.

6 month old boy with delayed motor development, weaker than other kids. Can't life his own head up prone and is unable to roll to the side. P.E. shows generalized hypotonia, and decreased DTR. Mutation of protein involved in assembly of snRNPs in motor neurons. Causes impaired function of what cellular elements?

Spliceosomes snRNPs are important parts of splicesomes (which remove introns from pre-mRNA in nucleus) Kid has: SMA (Spinal muscle atrophy) caused by mutation in SMN1 gene; results in impaired assembly of snRNPs in LMN. Infants have flaccid paralysis due to degeneration of anterior horn of spinal cord. Key: UMN = anterior horn of spinal cord.

PrP protein is normally fund in neurons with a Alpha-helical secondary structure. If the protein becomes Beta-pleated PrP, it becomes highly resistant to Proteases, thus causing tons of intracellular accumulation. Histo of brain?

Spongiform transformation of gray matter Key: Accumulation of Abnormal Prion proteins (Beta-pleated) causes resistance to proteases, and accumulation in brain! Hallmark of Prion Dx: Spongiform encephalopathy. (Ex: CJD, or Mad Cow Dx/Bovine Spongiform Encephalopathy). Later they form large cysts in brain.

Spongiosis?

Spongiosis is intercellular epidermal edema.

Staph aureus virulence factor?

Staph aureous has a Protein A virulence factor, an IgG binding outer membrane protein. Protein A virulence factor binds the Fc portion of IgG, thereby preventing phagocytosis, complement fixing, and opsonization.

Steepness of the Arterial tension curve?

Steepness of Arterial tension curve depends on solubility of the Anesthetic in blood. Partial pressure of gas rises very rapidly in blood with less soluble gas. This produces a much steeper curve.

65 y/o woman has poor memory, urinary incontinence, gait abnormalities. MRI shows enlarged ventricles, but with minimal cortical atrophy. Urinary incontinence in this pt is due to what?

Stretching of Descending cortical fibers Why? Bc Increased CSF will stretch the Descending fibers of Cerebral Cortex, which are normally supposed to descend down and inhibit Sacral Micturition center (S2-S4) to prevent involuntary peeing. Key: Old lady has Normal Pressure Hydrocephalus (Wet, Wobbly, Wacky) due to impaired absorption of CSF, causing Ventricular enlargement (swelling). Causes Urge incontinence (bladder fills with urine, pt doesn't even sense that bladder is full. Once pee totally fills bladder, causes reflexive involuntary peeing)

Stroke to what artery causes "Locked in Syndrome"?

Stroke (occlusion) to Basilar artery causes "Locked-In Syndrome" Why? Bc it damages the base of the Pons in the Brainstem (containing Corticospinal and Corticobulbar tracts) S+S: quadreplegia, fascial weakness, dysarthria (hard time speaking words), and Horizontal gaze palsy. Alert, but can't move or speak!

again, MCA stroke S+S?

Stroke causing occlusion of Right MCA: Causes CONTRALATERAL S+S: 1. Left sided hemiparesis/weakness/sensory loss of face 2. Loss of sensation, weakness in Left UEX. 3. Lower limb is spared 4. If Non-dominant (Right sided MCA) parietal lobe is damaged, can cause: Anosognosia (lack of insight) as well as Hemispace neglect syndrome of Left side (draw a clock, leaves the left side blank) 5. COnjugate gaze deviation toward the side of the actual stroke (Right side) 6. MCA stroke also causes Contralateral Homonymous Hemianopsia 7. Occlusion of Dominant (Left sided) MCA causes causes Right sided S+S as well as aphasia.

Subacute combined degeneration?

Subacute combined degeneration occur due to Vit B12 deficiency. Degeneration of dorsal columns, and lateral cerebrospinal tracts. Loss of 1. Dorsal column 2. Lateral CST 3. Anterior CST S+S: loss of position, vibration, and ataxia.

10 y/o boy recently immigrated to U.S. Has ataxia, myoclonus, and visual probs. Parents say he acts strange now. Brain biopsy shows RNA virus containing Hemagglutin. Dx?

Subacute sclerosing panencephalitis Key: SSP is a complication of Measles infection. It occurs several years after apparent recovery from Measles (Rubeola) Hallmark of Measles: cough, coryza, conjunctivitis, Koplik spots, maculopapular rash that SPARES palms and soles of feet--->Subacute Scelorising Panencephalitis within years. Measles: ssRNA (enveloped) Paramyxovirus. Contains Hemagglutin that allows cell surface adhesion

Shaken baby syndrome can cause what kind of hematoma?

Subdural hematoma SDH occur due to rupture of: Cortical bridging veins

Product that, when injected into local muscles of patients with relentless focal dystonias, causes temporary relief. What bacterial substance causes this?

Subterminal spore formation Key: Focal dystonias are uncontrolled muscle contractions. Giving this pt a shot of C. botulism (botox) can help cause flaccid paralysis. C. botulism is a Gram+, spore-forming anaerobic rods. Local botox injections can help treat focal dystonias, achalasia, and muscle spasms.

65 y/o man comes to office with tremors. Had idiopathic Parkinsons for 10 years, and treated with Levodopa/Carbidopa. However, for past few months, frequently develops "freezing" episodes, with bilateral resting tremors worsened, along with cogwheel rigidity, bradykinesia. Pt would benefit from high-frequency deep brain stimulation targeted at suppressing neuronal activity where?

Subthalamic nucleus Key: In P.D., long-term tx can cause fluctuating recurrence of S+S, especially when dose starts to taper off. "On" periods occur when S+S are controlled due to good serum drug levels. "Off" periods occur when drug wears off and S+S recur.

72 y/o man brought to ER bc of large involuntary movements of right arm. He was watching TV and his arm threw the remote across the room! Large-amplitude flinging movements in right arm What area of brain is affected?

Subthalamic nucleus Key: Pt has classic Lacunar stroke affecting the contralateral (left side) Subthalamic nucleus. The Subthalamic nucleus is key for modulation of BG movement. Lacunar stroke to subthalamic nucleus causes Contralateral hemiballism, with wild, flingy, movements on one side of body. Occurs due to chronic HTN/DM.

4 y/o boy brought to E.R. after seizure. Has fever, cough, runny nose past 2 days. Today he suddenly started shaking his arms and legs for like 30 seconds. In the E.R., his oral temp is 104.5F. Best next step?

Supportive care only!!! Key: Kid just has a Febrile Seizure, which are the most common neurological probs affecting kids, and are actually BENIGN consequences of a fever. So, kids with fever may have recurrent seizures, but will not develop epilepsy or anything. Tx: Just give them antipyretics for the fever (Acetomenophen or Ibuprofen) and make them comfy.

What regulates circadian rhythm?

Suprachiasmatic nucleus regulates Circadian rhythm. Processes light info from retina and relays in to other Hypothalamic nuclei and Pineal gland to secrete Melatonin.

1 y/o boy brought to ER with high fever, irritability, photophobia. Recently underwent ventriculoperitoneal shunt placement for congenital hydrocephalus. Passive flexion of neck results in spontaneous flexion of hips and knees. Blood culture shows coagulase-negative Staphylococcus. Virulence MOA?

Synthesis of an Extracellular polysaccharide matrix Key: Staph that Coagulase Negative is= Staph. epidermidis (producing layers of biofilm on prosthetic devices) Key: Biofilm is produced via extracellular polysaccharide matrix that encases and protects the bacteria.

Which drug causes: PRE-synaptic 5-HT and NE reuptake inhibition?

TCA (Amytriptyline) as well as SNRIs (Serotonin, NE repuptake inhibitors) will both inhibit reuptake of presynaptic Serotonin and NE. Ex: SNRIs (Venlafaxine)

What class of drugs inhibits reuptake of NE and 5-HT?

TCA inhibit reuptake of NE and 5-HT. Especially good choice for depressed pt with insomnia (like early morning awakening)

Side effect of TIA?

TIA lasts <24 hours. Usually only about 15 minutes. TIA can result in transient amaurosis fugax (transient monocular blindness)

Difference between temporal and spatial summation?

Temporal summation refers to sequential impulses from the same neuron over time. Spatial summation refers to simultaneous impulses from several different neurons.

Which part of brain is the MAJOR relay station for all Ascending sensory info?

Thalamus relays all Ascending sensory info

Small area of brain of a 54 y/o man shows neuronal shrinkage and intense Eosinophilic cytoplasm. If the patient actually survives, the area would look like what?

That area of the brain would have Glial hyperplasia. Key: Stem is describing irreversible neuronal damage (neuron shrinking and intense Eosinophilic cytoplasm) in brain. Key Key: Reactive Gliosis would occur in this area, eventually leading to: Glial hyperplasia.

Which nuclei control temp?

The Anterior and Posterior (A/P) nuclei of hypothalamus regulate temp control via ANS regulation. Key: 1. Anterior nuclei= controls cooling by TQ: inhibiting Adrenergic input, and allowing for stimulation of Cholinergic input to allow for vasodilation and proper sweating to cool off. So, destruction of Anterior nuclei causes hyperthermia (from lack o sweating) 2. Posterior nuclei regulates heat conservation via Adrengergic mediated vasoconstriction/shivering Destruction of Posterior nuclei= Hypothermia

Rupture in SAH?

The Anterior and Posterior communicating artery rupture at Circle of Willis (Berry) SAH often causes fever, and nuchal rigidity.

What form the Striatum of B.G.?

The Caudate and Putamen form Striatum of B.G. and function in motor activities. H.D. has loss of GABA-releasing neurons in Striatum (causes constant movement)

Which tract is a Descending motor pathway lying distal to pyramidal decussation?

The Lateral CST is a DESCENDING motor pathway, lying distal to the pyramidal decussation. Lateral CST is key for: highly skilled, voluntary movement

Area of decussation of STT?

The Lateral Spinothalamic Tract (STT) fibers decussate in: Ventral White Commissure. Key: Lateral STT transmits pain and temp from periphery to somatosensory cortex.

Which organelle is needed for Beta-oxidation of most fatty acids?

The Mitochondria is the queen of Beta-oxidation of fatty acids Also needed for 1. TCA 2. Ketogenesis 3. Beta-oxidation of fatty acids 4. ETC (ATP synthesis) 5. First 2 steps of the Urea cycle Note: The only fatty acids oxidized by peroxisomes are VLCFAs and Branched chain fatty acids. (Branched-chains have Odd # of Carbons)

Affects of the Puffer Fish toxin (Tetrodotoxin poisoning)

The Pufferfish toxin (Tetrodotoxin) poison will MOA: Inhibit Na+ influx into nerve cells. S+S: weakness, numbness, tingling, paresthesia in both face and extremities, and severe-ass HYPOTENSION. WORST: Pt's actually remain conscious, but paralyzed!!

Function of Golgi Apparatus?

The RER sends proteins to the--->Golgi apparatus. The G.A. allows postranscriptional modification of proteins from RER. Job of G.A. (after postranscriptional modification) is to package and distribute the proteins to their ultimate location (Ex: lysosomes, extracellular space) via membrane-bound vesicles

Risk of knicking what in Thyroid surgery?

The Recurrent Laryngeal nerve is a branch of the: Vagus nerve CN 10 It loops Below the Aortic Arch (on Left) and Below the Subcalvian artery (on Right) to provide motor innervation to Laryngeal muscles. Clinical: During Thyroid surgery, the Recurrent Laryngeal Nerve could be damaged due to its proximity to Inferior Thyroid artery Damage to Recurrent Laryngeal nerve could cause: hoarseness, inspiratory stridor, vocal cord paralysis

When would the brain vasodilate?

The brain has very little room for expansion since its so encased in the bone of the cranium. A small volume change allowing increased cerebral blood flow can really increase the ICP and cause brain compression! Main factors influencing cerebral circulation are: 1. Systemic BP 2. Arterial Blood Gas Levels (ABGs) Key: When BP is >150mmHg, there's too much flow to the brain, causing increased ICP. In contrast, BP<50 mmHg can cause cerebral hypoperfusion and ischemia to brain.

What does the femoral nerve innervate?

The femoral nerve innervates quadriceps (knee extension), and (hip flexion) Femoral nerve (L2-L4)

Landmark for pundenal nerve block?

The ischial spine is the landmark for pudendal nerve block during vaginal deliveries/perineum.

What happens to the shrunken and deeply Eosinophilic neurons?

The shrunken nuclei and Eosinophilic neurons undergo cell death and are then Phagocytosed by Microglia Microglia are the brain macrophages!

New Vaccine against N. meningitis contains a Meningococcal Pilus Antigen. Vaccine admin to lab mice causes Monoclonal Ab to bind against pilus protein antigens. These antigens impair normal pilli function. These Ab of the vaccine directly interfere with what process on N. meningitis??

These Ab directly inhibit the ability of the N. meningitis protein Antigens to adhere to nasopharynx epithelial cells. Key: The Pilli of N. meningitis attach to epithelial surfaces. Ab that inhibit these pili will thereby inhibit pilus-mediated attachment to mucosal epithelium and nasopharynx.

Which Vitamin participates in Glucose metabolism?

Thiamine (VitB1) participates in Glucose metabolism Key: VitB1 Thiamine is a cofactor for: 1. Pyruvate dehydrogenase (converts pyruvate at end of Glycolysis--->into Acetyl CoA which enters the TCA) 2. Alpha-Ketoglutarate Dehydrogenase (in TCA) 3. Branched-chain alpha-ketoacid dehydrogenase (needed for catabolism of branched chained amino acids like Leucine, Isoleucine, Valine) 4. Transketolase (enzyme found in Pentose Phosphate Pathway) which is needed to convert Ribulose 5-P (derived from Glucose) into Glyceraldehyde 3-P Tx: Stat Thiamine supplements with Glucose infusion

What causes Beriberi?

Thiamine Vit B1 deficiency causes 1. Ber1ber1 2. Wernike-Korsakoff syndrome

Blood/gas partition coefficient?

This also measures solubility. However, unlike AV conc gradient, Higher Blood/gas partition coeff just means that anesthetic is absorbed to a greater extent by BLOOD, and has slower onset of action.

67 y/o man with vesicular rash on a red base, extremely tender, itchy. Has Varicella IgG Ab.What does he have?

This old man is having reactivation of VZV (bc he has IgG Ab, which means he had Chicken pox in past, latent in DRG, and now has Shingles) Note: VZV is chicken pox Reactivation is Herpes Zoster (Shingles)

What is threshold determined by?

Threshold refers to membrane potential value needed to initiate an action potential. Determined by: Voltage-gated Sodium channels present in membrane.

Function of Median nerve?

Thumb flexion and opposition Flexion of wrist and middle digits (2+3), ABduction Helps with forearm pronation

24 y/o man brought to ER after falling off roof and injuring right shoulder. Closed reduction, internal fixation of shoulder is performed via IV Propofol. However, he becomes alert a few minutes after admin. Why did he have rapid recovery from the anesthesia?

Tissue redistribution of Propofol Key: Propofol and other Lipophilic drugs easily diffuse across membranes, and accumulate in tissues that receive a ton of blood flow (brain). Bc they rapidly diffuse across membranes, they have a rapid onset of action. These lipophilic drugs are then quickly redistributed to other tissues/organs receiving less blood flow (fat, muscle tissue). Redistribution accounts for very rapid termination of action.

What CN allows tongue protrusion?

Tongue protrusion is via motor efferent fibers of CN 12 (hypoglossal)

What does dose-dependent mean?

Too much Phenytoin admin will overwhelm CytP450 metabolism machinery, casuing buildup and toxicity of Phenytoin.

Cause of pts S+S?

Toxoplasmosis Key: Ring enhacing lesions on MRI are hallmark for HIV pts infected with Toxoplasmosis. Hey has oral thrush (CD4 count<200), cervical/inguinal lymphadenopathy, and brain lesions causing seizures.

65 y/o with acute onset slurred speech. Right-sided weakness. Right sided lower face droop and +Babinski's sign. Decreased motor strength on right compared to left. Also has Dysmetria (over/undershoot of target) and Dysdiadochokinesia (can't perform rapid movements). What CN exits brainstem at level of patient's stroke?

Trigeminal CN ??

man has double vision. hard time walking down the stairs, and sees duplicates of every step. No problem walking upstairs though. Lesion to what structure?

Trochlear nerve Key: Trochlear nerve innervates the: Superior oblique (SO), which causes eye to "depress down while adducted" Key: CN4; Trochlear nerve palsy: causes "Vertical Diplopia" that worsens when pt looks down or towards nose (walking downstairs, up-close reading, etc) Compensate by tilting head AWAY from affected eye.

What is in milk before bedtime that helps me sleep?

Tryptophan is in milk before bedtime.

26 y/o female lacks menstrual period last 3 months. Bilateral milky nipple discharge. Takes meds that prevent her from "hearing voices." Interruption of what pathway?

Tuberoinfundibular pathway Key: Pt is taking an atypical antipsychotic, inhibiting Dopamine, allowing Prolactin to be secreted. Key: Dopamine pathways include 1. Mesolimbic and Mesocortical (regulate cognition and behavior) 2. Nigrostriatal pathway (regulates voluntary movements) 3. Tuberoinfundbular pathway (inhibits release of Prolactin) Key: disruption of tuberoinfundibular pathway by blocking D2 receptors allows hyperprolactinemia and amenorrhea.

Generalized Tonic-Clonic Seizures

Typically begins with a tonic phase contraction of ALL muscles for 10-15 seconds leads to a fall "like a tree" The clonic phase follows the tonic phase rhythmic jerking of the bilateral extremities There are postictal deficits

Precursor for Thyroxine, NE, Epi, DA, and Melanin?

Tyrosine Key: Tyrosine--->Melanin (via Tyrosinase) defective in Albinism.

What is Ubiquitin?

Ubiquitin is a protein that undergoes ATP-dependent attachment to other proteins, labeling them for destruction. These Ubiquitin-labeled proteins enter Proteasomes and are degraded into small peptides. Defect in ubiquitin-proteasome system causes neurodegenerative disorders like Parkinson's Disease and Alzheimer's Disease.

4 day old baby girl brought to office. She is exclusively breast fed (clue AF!!) She has increasing difficulty feeding over past 24 hours. "Too sleepy" to be fed and vomits a lot. Her brother died as an infant due to "low sugar in his blood." She is dehydrated, and UA reveals elevated MMA (Methylmalonic Acid). Labs of this pt?

Urinepropionic Acid: High Serum Glucose: Low Urine Ketones: High Serum Ammonia: High Key: This baby girl has classic Methylmalonic Acidemia (A.R.) due to: deficiency in Methylmalonyl-CoA mutase

First branch of CNV?

V1= opthalmic nerve. Passes through Superior orbital fissure.

V2 (Maxillary) branch of trigeminal CN?

V2 (Maxillary) branch of trigeminal nerve supplies cheeks, and pharyngeal palate, and of course the maxillary sinuses!

Action of V2 branch of CN5?

V2 branch is the Maxillary nerve. V2 exits skill via Foramen Rotundum

Mutation on Chrom 3?

VHL disease mutation on Chromosome 3 S+S: cysts in kidney, RCC, cysts in brain Cerebellum (Hemangioblastomas), cysts in retina

18 y/o with fever, headache, N/V, myalgias. Confused, difficult to arouse. Nuchal rigidity and purpuric rash on LEX. CSF shows elevated leukocyte count, high protein and LOW glucose. Vaccine that would have prevented this?

Vaccine would contain a capsular polysaccharide Pt has N. meningitis. Immunity can occur by vaccine providing antibodies against the polysaccharide capsule. Note: Polysaccharide capsules normally protect bacteria from phagocytosis.

7 day old neonate has lethargy, vomiting, poor feeding. Mom exclusively breastfeeds. Infant is somnolent (sleepy), dehydrated, and has decreased muscle tone. Labs: Elevated Anion Gap, Ketosis, and Hypoglycemia. Also, elevated Proprionic acid due to defect in conversion of Propionyl-CoA to Methylmalonyl-CoA. Neonate can't use what amino acid to make energy?

Valine Key: Catabolism of essential amino acids (Valine, Isoleucine, Methionine, and Threonine) as well as Odd-chain fatty acids--->allows generation of Propionyl-CoA. Next, Proprionyl-CoA is converted to Methylmanolyl-CoA (via Propionyl-CoA carboxylase) Methylmanolyl-CoA then makes--->Succinyl-CoA--->enters TCA for ATP (energy production) Key: This baby has high levels of Proprionic acid bc of lack of Propionyl-CoA carboxylase. Causes excess accumulation of Proprionic acid (and severe Metabolic acidosis) Also has hypoglycemia, lethargy, and ketosis, Tx: Low-protein diet containing minimal protein (low Valine, low Isoleucine, Low Methionine, low Threonine)

Side effect of taking Valproate or Lithium during Pregnancy for moms with Bipolar Disorder?

Valpraote during pregnancy for Bipolar Dx or Epilepsy can cause x10-20 fold risk of neural tube defects due to impaired folate metabolism. Lithium causes Ebstein's anomaly.

Pt has episodes where she stares into space, and also sometimes loses consciousness and starts shaking and jerking. Best long term tx?

Valproate is #1 DOC for both: 1. Absence seizures 2. Generalized Tonic-clonic seizures Note: Ethosuxamide sucks bc it only treats Absence seizures.

15 y/o boy has jerking movements in both arms. Aggravated by sleep deprivation. Fam hx of seizure dx. Best tx?

Valproic Acid Key: Pt probs has Myoclonic seizures (form of idiopathic Generalized epilepsy due to fam hx.) S+S: brief, involuntary jerky movements that start in teens. No LOC. Tx: Broad-spectrum anticonvulsants (Valproic Acid) treat most seizure types

What would be the end result of Vascular Dementia?

Vascular Dementia would cause sudden stepwise decline in patients with CV risk factors and ischemic stroke. Would result in a Glial scar.

Last resort tx for Treatment-resistant Epilepsy?

Vigabatrin MOA: irreversible inhibitory of GABA transaminase. Prevents breakdown of GABA in CNS.

35 y/o female hospitalized with psych illness and has weird dietary habits. She has papilledema, dry skin, and hepatospleomegaly. Cause?

Vit A OVERuse Vit A toxicity overuse causes intracranial HTN, skin changes, and hepatospleomegaly. Papilledema is a sign of high intracranial pressure! Note: Can't use Vit A (Retinoic Acid) when pregnant cuz it's teratogenic.

Vit B2 deficiency?

Vit B2 (Riboflavin) deficiency causes: cheliosis, stomatitis, glossitis, dermatitis, corneal vascularization

Niacin deficiency

Vit B3 (Niacin defiency) Pellagra (3 D's) 1. dementia 2. Dermatitis 3. Diarrhea

16 y/o boy has gait instability, dysmetria (under/overshoot target), dysarthria for past few years. No longer can play basketball. Bilateral motor weakness, loss of DTR, loss of proprioception and vibration. Cause?

Vit E deficiency Key: Vit E is a lipid soluble vitamin (ADEK). Vit E is a crucial Anti-oxidant. Deficiency of Vit E mimics Vit B12 deficiency (without the hypersegmented PMN). S+S: ataxia (degen of Spinocerebellar tract), loss of position, vibration sense (DCT degen., loss of DTR (degen of peripheral nerves)

Old immigrant from Middle East has numbness, tingling in hands and feet. Has pulmonary T.B. and currently is on Rifampin and INH. Now has sensory ataxia, and decreased pain sensation in distal extremities. Cause?

Vitamin deficiency Key: INH is structurally similar to VitB6 (Pyridoxine) Therefore, INH tricks body into thinking it has way too much VitB6, causing VitB6 urinary excretion. This leads to INH neuropathy (peripheral neuropathy with n,t,p) Tx: prevent these S.E. from occurring in the first place by making sure to give pt on INH some VitB6 Vitamin supplements (like prophylaxis) So, this pt's peripheral neuropathy is directly due to Vitamin deficiency, NOT drug-mediated toxic neuron loss.

What's St Johns Wart?

Warfarin can interact with Dt Johns Wart (which is just a weird OTC medicinal herb that's anti-inflammatory and antidepressive) St Johns Wart induced CYP450 enzymes, resulting in way too much Warfarin metabolism, decreasing the drug level and effect, inhibiting effective anticoagulation. Note: CytP450 inhibitors would cause Warfarin buildup and toxicity, ultimately causing bleeding complications.

Weak Inferior gluteal nerve?

Weak Gluteus maximus So: Weak extension and external rotation of thigh at hip. Hard time rising from seated position, or climbing stairs.

Damage to Median nerve?

Weakness in wrist flexion Normal function of Median nerve: 1. Wrist Flexion 2. Thumb opposition (ability of thumb to touch other fingers one by one) 3. Wrist Pronation

Hemorrhage and necrosis in mammillary bodies is found in?

Wernicke's Encephalopathy due to Vit B1 (Thiamine) deficiency Thiamine needed in glucose metabolism. Lack of Glucose utilization is especially pronounced in CNS.

Where is the Wernicke's area located?

Wernicke's area is located in the primary auditory cortex (Located in Posterior Temporal Lobe) Both Wernicke's and Brocca's areas are located in Dominant (Left) side of brain, and supplied by MCA.

What is irreversible in Wernicke's encephalopathy?

Wernicke's encephalopathy: Alcoholics with lack of Vit B1 (Thiamine) have CAN Must administer rapid Thiamine IV infusion. Otherwise, they develop Korsakoff's Syndrome: Confabulation and Anterograde amnesia (permanent)

What happens when Epi binds to Alpha1 adrenoreceptors?

When Epinephrine binds to Alpha-1 Adrenoreceptors, it results in activation of IP3 pathway. IP3 is (Inositol Triphosphate) IP3 pathway allows increased intracellular Ca2+ activity via release of Ca2+ from storage organelles.

When would dependence and addiction become more at risk?

When the drug has a shorter half life (shorter high) so constantly chase the high. Ex: Triazolam is a Benzo with a half-life (T1/2=2 hrs) of only 2 hrs. Increased risk of dependence and addiction. Long acting Benzos like Flurazepam, are lesser risk of addiciton than short acting Benzos.

inhertience pattern of DMD or BMD?

X-linked recessive (so affects boys) Mutation in gene that encodes for Dystrophin muscle protein causes either complete absence of Dystrophin protein, DMD, or decreased synthesis, BMD)

Duschenne muscular dystrophy

X-linked recessive, affects boys. Due to: Frameshift mutation of Dystrophin gene.

Is MAC an intrinsic property?

Yes! MAC is a totally intrinsic property, meaning it DOES NOT depend on type of surgery, duration of anesthesia, sex, height, weight,etc IT DOES depend on body temp, and also MAC decreases with increased age of pt (meaning potency of drug is higher the older you are)

Do Mitochondrial diseases affect both males and female offspring with equal frequency?

Yes! Mitochondrial diseases affect both male and female offspring with equal frequency (100%) However, there are variable degrees of severity.

Zapping what area of Hypothalamus will cause "food seeking behavior"?

Zapping the VentroMedial nucleus Of Hypothalamus causes constant hunger, and weight gain. Fat kid yo. Why? Key: Hypothalamus is the key region to Homeostasis. the VM nucleus is center for satiety (responds to Leptin) Lesion will lead to lack of response to satiety, constant hyperphagia, and obesity.

70 y/o female has sleep problems. She wants some sleeping aid that she won't become addicted to. What med?

Zolpidem Key: Zolpidem is short-acting hypnotic sleep aid, that's actually chemically NOT related to Benzos. Much lower risk of tolerance or dependence. MOA: Even though it's structurally different, Zolpidem has same MOA as Benzos. Binds to same GABA-rec as Benzos and enhances GABA inhib activity on CNS.

Partial (focal) seizure

abnormal neuronal activity in a localized (focal) part of the brain

Generalized seizures

abnormal neuronal activity in both hemispheres of the brain impaired consciousness generalized tonic-clonic (grand mal) seizures is the most common type of generalized seizure

Ilioinguinal nerve?

accompanies Spermatic cord through superficial inguinal ring. Ilioingunal ring supplies sensation to upper and medial thigh and external genitalia.

Side effects of Carbamazepine?

aplastic anemia, so must moniter CBC closely, like Clozapine. Also, Carbamazepine is a CytP450 inducer, which causes too much increased metabolism of other meds, thereby reducing their effectiveness.

Schwannomas

benign tumors arising from CN8 at cerebellopontine angle.

Schwannomas?

benign tumors of CN8 at cerebellopontine angle (vestibular schwanomma). Causes sensoneural hearing loss and tinnitus.

Where does blood accumulate in SAH?

between Arachnoid and Pia Note: SAH can cause nuchal rigidity bc it irritates meninges.

Vesamicol?

blocks vesicular Ach transporter, (VACHT), which inhibits Ach from even entering vesicles in presynaptic nerve terminal.

Subarachnoid hemmorhage?

blood between arachnoid and pia matter. SAH occur as "worst headache of my life" due to rupture of berry aneurysms or AVM of PCA, ACA, or MCA.

Temporal bone fractures can impair what CN?

can impair CN7 (controls motor function of face), can be hurt in Temporal bone fx. (lateral head trauma/Zygomatic bone fx)

Brainstem lesion?

causes "crossed signs" 1. IPSILATERAL CN palsy 2. CONTRALATERAL Hemiparesis can also cause vertigo, ataxia

Lesion to Parietal Association Cortex (in NonDominant Hemisphere)

causes Hemispace Neglect Syndrome Contralateral neglect, anosognosia (lack of awareness of illness), and construction apraxia (can't draw objects asked to draw) Ex: Draw a clock without contralateral side.

Atrophy of Caudate nucleus?

causes Huntington's Disease, with chorea (low-amplitude) movements, dementia, depression.

Degeneration of Substantia Nigra?

causes loss of DA, and Parkinson's Disease (TRAP)

Foramen Spinosum houses?

contains Middle meningleal artery and vein, the Meningeal (recurrent) branch of V3 which supplies Dura and has SNS fibers.

damage to what nerve would cause weak respirations?

damage to phrenic nerve would decrease RR.

Cause of X-linked Adrenoleukodystrophy?

defect in transport of VLCFAs into Peroxisomes for oxidation. Lack of oxidation causes buildup of fatty acids in tissues and nervous system. Causes neurological deterioration, adrenal insufficiency, and death.

Pompe's Dx?

deficiency in: alpha-1,4-Glucosidase S+S: Cardiomegaly (think Pompe's Volcano being HUGE like Cardiomegaly), as well as severe Hypotonia

What is variable expressivity?

difference in severity in specifically Autosomal DOMINANT disorders. Ex: Marfan's syndrome patient may just be tall, while another Marfan's patient could be tall, have aortic root dilation, and lens dislocation.

Frontal lobe lesion?

dis-inhibition, probs planning/organizing, personality changes

Cause of homonomyous hemianopia?

due to cut of contralateral optic tract. Note: optic tracts links the optic chiasm to the lateral geniculate body. Ex: Cut of left optic tract causes contralateral right homonymous hemianopsia.

When would Vit E deficiency occur?

fat malabsorption Abetalipoproteinemia Bc deficiency will cause lack of Vit E's antioxidant powers, you will have more free radical damage of cell membranes (RBC)

MS brain biopsy would show?

focal demyelinated plaques with relative axonal sparing

How to assess comprehension?

follow multi-step commands

Man has stab injury in bar fight. Penetrating wound over left neck and shoulder. Feel dizzy, BP is 100/60. Given IV fluids stat. Transection of this nerve labeled 8---> what function is impaired?

he damaged his Radial nerve, so he'd have impaired wrist Extension Key: Radial nerve is Largest branch of brachial plexus. Provides Extension of Upper Limbs below shoulder, and Wrist Extension

Cause of Epidural Hematoma?

head trauma (Natasha Richardson) Due to: Temporal bone fracture and tear of: Middle Meningeal artery

MOA of 1st-generation Antihistamines?

help relieve S+S of seasonal allergies by inhibiting release of Histamine from mast cells (rhinorrhea, itchy eyes, hives, urticaria,etc) However, 1st-generation Antihistamines are also Anticholinergic. Note: 1st-gen also cross BBB easily into CNS, causing drowsiness and cognitive dysfunction.

Use for Propofol?

highly lipophilic anesthetic used for both induction and maintenance of general anesthesia ans procedural sedation. IV bolus admin, action in <30 seconds, lasts only <10 minutes.

Locus Cereleus

houses NE/Epi

What are neurofibrallary tangles of tau?

hyperphosphorylated tau protein. Cause senile neuritic plaques composed of Beta-amyloid. Beta-amyloid deposition in walls of Cerebrum, causing Amyloid angiopathy. Alzheimer's Dementia

Hypervariable pilli are characteristic of?

hypervariable pilli are found in: Neisseria meningitis, and N. gonorrhea. Makes vaccination really really hard

When does Pseudotumor Cerebri occur?

idiopathic intracranial HTN causes elevated ICP occurs in young, obese women with headache and Papilledema, can also cause visual loss

Pathological process involving entire optic nerve?

if it involves the ENTIRE optic nerve, it leads to monocular blindness

Woman has gas pain and becomes preoccupied with worry that she has CRC? Dx?

illness anxiety disorder

Alpha-1

in peripheral vasculature Eye MOA: Increases BP, contracts internal urethral sphincter, causes mydriasis

MOA of P-TU and Methimazole?

inhibits TH synthesis by suppressing the: Iodination and coupling of Tyrosine

Craniopharyngiomas

kids located in Suprasellar region Can be "calcified" S+S: visual distrubances (Bitemp. hemianopia) due to endocrine problems (hypopituitarism)

62 y/o man with Left arm clumsiness. P.E. shows motor weakness in left arm and leg. Slurred speech, drooping of left lower face. Lesion to where in cross section of brain?

lesion to Right sided (Contralateral) Intenral Capsule Internal capsule strokes cause Pure Motor weakness, affecting the contralateral arm, leg, and lower face.

Pupillary light reflex:

light enters one eye, retina generates signal that's transmitted along optic nerve bilaterally to pretectal nucleus (located in Superior Colliculus). From there, fibers are projected to ipsilateral and contralateral Edinger-Westphal nuclei.

Ependymal cells?

line ventricles in CNS, and make up the: Choroid Plexus Ependymal cells Produce CSF

Lesion to frontal cerebellar cortex S+S?

loss of executive function and response inhibition. Personality changes, inability to organize a plan, disinhibited.

Femoral nerve injury leads to?

loss of flexion of hip (thigh), loss of knee extension (quads) Loss of patellar reflex

24 y/o college student with excess daytime sleepiness. Also has episodes of sleep paralysis where his knees would buckle during episodes of laughter or crying. What is low in his CSF?

low Hypocretin-1 (Orexin-A) promotes wakefulness and inhibits REM sleep-related phenomenon.

Day 3-7 days s/p I.S.

macrophages/microglia clear debris via phagocytosis

Glioblastoma

malignant tumor with butterfly shape

What nerve innervates Thenar eminence?

median nerve

Function of Anterior Hypothalamus?

mediates heat dissipation; destruction leads to Hyperthermia

What things have psammoma bodies?

meningioma Papillary thyroid carcinoma Mesothelioma

Acute hemorrhage of Cerebellar vermis?

midline lesion causing: Truncal ataxia, vertigo, nystagmus

Down Syndrome "mosaicism"

milder form of Down Syndrome. Some cells, but not all have an extra copy of Chromosome 21.

LMN lesions

muscle Atrophy fasciculations loss of DTR

What is Dantrolene?

muscle relaxant, used for MH. Acts on RNR to prevent release of any more Ca2+ into cytoplasm of muscles.

Cocaine overdose S+S?

mydriasis, tachycardia, tachypnea, HTN.

Ulnar nerve damage?

normally Ulnar nerve will 1. allows Finger ABduction and ADDuction 2. Thumb ADDuction 3. Wrist Flexion and ADDuction towards body

Neuroblastoma S+S?

occurs around age 2y/o Homer-Wright Rosettes P.E. will show Retroperitoneal mass (palpable mass), HTN (Due to excess Epi/NE) Can cause a Parneoplastic effect leading to Opsoclonus Myoclonus Syndrome (crazy eye movements) Urine: Increased excretion of Urine Catecholamines Cause of Neuroblastoma: N-myc gene amplification (so higher the # of N-myc copies, the worse the prognosis)

Non-communicating Hydroceph?

occurs due to an OBSTRUCTION, which impeded CSF flow Ex: Arnold-Chiari, or tumors

When does a Transtentorial (Uncal) Herniation occur?

occurs due to an intracranial mass (tumor, or hemorrhahe, or cerebral edema) Causes compression of CN 3 and PCA. S+S: fixed, dilated pupil on same side of lesion = #1MCC sign of red flag

Cause of Epidural hematoma?

occurs due to tear of: Middle Meningeal artery. Due to a temporal bone fracture, located between Bone and Dura mater. Hallmark: Lucid interval, followed by LOC. (Natasha Richardson!)

Irreversible neuronal injury

occurs due to transient but profund Hypoxia, ischemia, or toxic injury that all may lead to loss and damage of neurons Irreversible neuronal injury: Shrinkage of neuronal body, Intensely eosinophilic cytoplasm, nuclear pyknosis, loss of Nissl RER

Hemorrhagic necrosis

occurs is tissues with dual blood supply like the lungs (supplied by both pulmonary and bronchial arteries). Hemorrhagic infarcts of CNS result from reperfusion injury.

What is a Transtentorial (Uncal) herniation?

occurs when Medial temporal lobe (aka Uncus) herniates through gap between falx cerebri and tentorium cerebelli. Causes: ipsilateral mass lesion, like a tumor, or hematoma, or hemorrhage. An Uncal herniation will compress 1. CN3 causing fixed, dilated, down and out pupils ipsilaterally. 2. can also cause: Ipsilateral PCA compression: causing CONTRALATERAL homonymous hemianopsia with macular sparing 3. CONTRALATERAL hemiparesis or/and IPSILATERAL hemiparesis

Beta-1

on heart MOA: Increases HR, increases contractility, and conductance (increases inotropy and chronotropy)

Beta-2

on peripheral vasculature, bronchi, Uterus MOA: Vasodilation (decreases BP), Bronchodilation, uterine relaxation (tocolysis)

Sensory of tongue?

pain, temp, touch of tongue is via Anterior 2/3: V3 (Mandibular) Posterior 1/3: CN 9 (Glossopharngeal nerve) and Vagus CN 10

Orientation

provide location, date, name.

Genitofemoral nerve?

provides sensation to upper anterior thigh and motor function to genitalia (Creamsteric Reflex in men= Genitofemoral nerve)

Femoral nerve?

provides sensation to upper thigh and inner leg Femoral nerve innervates muscles that EXTEND knee, flex the hip.

Encephalocele

rare neural tube defect that occurs due to protrusion of brain and meninges through abnormal opening in skull.

16 y/o, eye injury at baseball game in Right eye. Diplopia, worsens when he looks up, limited vertical movement of Right eye. CT shows Orbital fracture. What other S+S?

reduced sensation over Right upper lip Key: Fracture to Orbital floor is common in frontal trauma. The infraorbital nerve runs along Orbital floor in a "groove" in Maxilla, before exiting skull. Damage to orbital floor can impair infraorbital nerve, causing paresthesia of cheek, upper lip, and also damage IR mucle (problem with vertical gaze) Key: Orbital floor is made up of the Zygomatic bone and Maxilla. The Infraorbital nerve is a continuation of the Maxillary nerve (V2)

MOA of Valproic acid?

reduces abnormal electrical activity in brain by blocking NMDA receptors and affecting Potassium current.

What is anticipation?

refers to earlier onset and increased severity of genetic disorder in subsequent generations. Often seen in trinucletoide repeats (Huntington's Disease, Fragile X Syndrome)

Suprachiasmatic nuclei

regulates Circadian Rhythm in hypothalamus Key: it's located right above the Optic chiasm, so it get light input from Retina, and relays light info to rest of Hypothalamus and Pineal gland.

Early fibroblast migration to area of injury?

results in substitution of necrotic tissue with a fibrous scar

What the hell is a Charcot-Bouchard Aneurysm?

rupture due to Chronic HTN. Causes Hemorrhage involving deep brain structures like the: Basal ganglia, Cerebellum, Thalamus, Pons.

What CN allows for salivation?

salivation from submandibular and sublingual glands mediated by PNS fibers of CN 7

Fibrinoid necrosis?

seen in walls of blood vessels affected by classic Vasculitis with immune complex deposition (Polyarteritis nodosa), malignant HTN, DM, etc.

What is genetic imprinting?

selective inactivation of paternal or maternal alleles. Ex: Prader Willi vs Angelman Syndrome

Lateral femoral cutaneous nerve?

sensation to lateral thigh.

short-term vs long term memory

short term: recall 3 unrelated words after 5 minutes. long term: provide details of significant life events

Note: highly soluble anesthetics dissolve too much in blood, so more of it must be added in order for blood to become saturated Saturation occurs when the P.P. of the gas rises in blood till where it reaches its P.P. in the inspired gas. At this pt, no more of the anesthetic can be dissolved in blood, so blood is saturated. Speed at which blood saturates depends on:

solubility of gas

Neonate has respiratory distress taken to NICU. Has underdeveloped mandible and hypoplastic zygomatic process. Gene mutation that resulted in abnormal development of 1st and 2nd pharyngeal arches. What other structure would be abnormal?

stapes 1st pharyngeal arch: CNV: mandibular, maxillary, lateral and medial pterygoid, malleus and incus. 2nd pharyngeal arch: CN7; stapedius, stylohyoid (lesser horn), stapes, platySma.

occlusion of central retinal artery?

sudden, painless, complete loss of vision in affected eye.

taste of tongue

taste buds Anterior: CN 7 Posterior: CN 9 (Glossopharyngeal) and Vagus CN 10 (especially taste buds down larynx and esophagus)

What are the 3 signal pathways utilized by the ANS?

the ANS uses three different signal pathways: 1. cAMP 2. IP3/DAG 3. Ion channels Key: Nicotonic receptors are: Ligand-gated ion channels, which open after binding to Ach. This then results in IMMEDIATE influx of Na+ and Ca2+ into cell, and outflux of K+ out of cell.

How long is it normal for infants to have +Babinksi:

up to 12 months +Babinksi (stroke plantar surface of foot) causes Dorsiflexion/toe spreading Adults plantar stroking should cause: Plantarflexion UMN "Pyramidal" signs can cause: +Babinski, spactisity, hyperreflexia, muscle paralysis and weakness

Vagus nerve lesion S+S?

vagus nerve (CN 10) lesion can cause ipsilateral paralysis of soft palate pharynx, larynx. Produced hoarseness, dysarthria, dysphagia, and loss of gag reflex. The uvula would deviate AWAY from side of lesion due to weakness of Levator Veli Palatine.

visual spatial vs executive function

visual spatial: draw intersecting pentagons executive function: draw a clock oriented to time requested

S+S of von Gierke's Disease?

von Gierke's Dx is a classic Glycogen storage disease due to deficiency in: Glucose-6-Phosphate S+S: hepatomegaly, hypoglycemia, seizures, lactic acidosis

MRI shows young pt with angiomatous lesions in cerebellum. Cystic mass in Right kidney on abdominal U.S. Dx?

von Hippel Lindau Dx (MEN2A/MEN2B) Note: VHL dx is an A.D. disorder. hemangioblastomas in brain, as well as cysts in liver, kidney, brain. Increased risk of BILATERAL RCC.


Ensembles d'études connexes

The Nature of Probability and Statistics

View Set

Chapter 17: Schizophrenia Spectrum Disorders

View Set